Sunteți pe pagina 1din 55

Copyright © 2014 Delhi Academy of Medical Sciences, All Rights Reserved.

1/55
Test Information
Test Name MD/MS-SWT-Pathology-2019 Total Questions 200

Test Type Examination Difficulty Level Difficult

Total Marks 800 Duration 120minutes

Test Question Language:- ENGLISH


(1). All are mesothelial markers used to identify malignant mesothelioma except

a. Calretinin

b. WT1

c. D2-40

d. CEA

Solution. D
Mesothelial markers- Calretinin, CK 5/6, Thrombomodulin, WT1, Mesothelin, D2-40, h-Caldesmon, vimentin
Epithelial markers- CEA, Ber-EP4, BG-8, MOC-31, CD 15, TTF1

Correct Answer. d

(2). In which of the following areas in the cervical epithelium is the risk of HPV infection the maximum?

a. Mature squamous epithelium

b. Immature squamous epithelium

c. Stroma

d. Columnar epithelium

Solution. B Immature squamous epithelium


Reference: Robbins and Cotran’s Pathological Basis of Disease, 9th edition and Basic Pathology, 10th edition
Immature squamous metaplastic epithelial cells in the transformation zone are most susceptible to HPV infection, and as a result this is
where cervical precursor lesions and cancers develop.

Correct Answer. b

(3). All of the following is/are true about extrinsic pathway of apoptosis except

a. Engagement of plasma membrane death receptors

b. Type I TNF receptor is prototype

c. Caspase 9 is classically involved

d. Mechanism of apoptosis of virus infected cells

Solution. C
Caspase 8 and 10 (in humans) are involved in the extrinsic pathway

Correct Answer. c

(4). Afibrinogenemia is characterised by all except

a. Normal platelet count

b. Raised bleeding time

c. Normal PT and aPTT

d. Raised Thrombin time

Copyright © 2014 Delhi Academy of Medical Sciences, All Rights Reserved. 2/55
Solution. C

Correct Answer. c

(5). All are autosomal recessive coagulopathies except

a. Bernard Soulier syndrome

b. Glanzmann Thrombasthenia

c. Grey platelet syndrome

d. Von Willebrand disease

Solution. D
Reference: Consultative Hemostasis and Thrombosis, 4th edition (2019)

Correct Answer. d

(6). Which of the following combinations is true for myotonic dystrophy?

a. CTG repeats on chromosome 20

b. CTG repeats on chromosome 19

c. CAG repeats on chromosome 20

d. CAG repeats on chromosome 19

Solution. B

Correct Answer. b

(7). The core criteria used in the prognostic evaluation of Neuroblastoma are all except

a. Age

b. DNA ploidy

c. Histology

d. TRKA expression

Solution. D
Age, stage, histology,, MYCN status, and DNA ploidy are currently the “core” criteria used for the purposes of formal risk stratification
and therapeutic decision

Correct Answer. d

(8). The most important prognostic indicator for Wilm’stumor is

a. P53 mutations

b. Stage

c. Histology

d. Age

Solution. C
Evidence of anaplasia on histology is the most important prognostic indicator of Wilm’stumor.

Correct Answer. c

(9). Which of the following is false about lipofuscin?

Copyright © 2014 Delhi Academy of Medical Sciences, All Rights Reserved. 3/55
a. Injurious to cell

b. Perinuclear in location

c. Lipid peroxidation

d. Yellow brown granular pigment

Solution. A
Lipofuscin is not injurious to the cell or its functions. Its importance lies in its being a telltale sign of free radical injury and lipid
peroxidation. The term is derived from the Latin ( fuscus, brown), referring to brown lipid. In tissue sections it appears as a yellow-brown,
finely granular cytoplasmic, often perinuclear, pigment.

Correct Answer. a

(10). Almost all inborn errors of metabolism show the following inheritance

a. Autosomal dominant

b. Autosomal recessive

c. X linked dominant

d. X linked recessive

Solution. B

Correct Answer. b

(11). Lineage specific T cell marker is

a. CD 1

b. CD 2

c. CD 3

d. CD 4

Solution. C
CD 3 is a part of T cell receptor complex and is considered to be lineage specific for T cells

Correct Answer. c

(12). Following is the histopathology photograph of the lung biopsy from upper left lobe of a patient with occupational exposure and fibrosing
lung disease. Which of the following is false about this condition?

a. No increased risk of cancer

b. Increased risk of tuberculosis

c. Both free radical injury and activation of macrophages are involved in the pathogenesis

d. Usually involves the upper lobes

Solution. A
The photograph shows dense collagenous nodules indicative of silicosis. Silica occurs in both crystalline and amorphous forms, but
crystalline forms (including quartz, cristobalite, and tridymite) are much more fibrogenic. Of these, quartz is most commonly implicated.
Silicosis is slow to kill, but impaired pulmonary function may severely limit activity. It is associated with an increased susceptibility to
tuberculosis. This may be because crystalline silica inhibits the ability of pulmonary macrophages to kill phagocytosed mycobacteria. The
onset of silicosis may be slow and insidious (10 to 30 years after exposure; most common), accelerated (within 10 years of exposure) or
rapid (in weeks or months after intense exposure to fine dust high in silica; rare). Patients with silicosis have double the risk for
developing lung cancer.

Correct Answer. a

(13). All of the following are true about AL Amyloidosis except

Copyright © 2014 Delhi Academy of Medical Sciences, All Rights Reserved. 4/55
a. Heart is the most common organ involved

b. Renal involvement presents as proteinuria

c. Macroglossia is a pathognomonic sign

d. Raccoon eye sign can be seen

Solution. A
• The kidneys are the most frequently involved organ- nephrotic range proteinuria
• Heart is the second most common involved organ- leading cause of mortality
• Subendocardial involvement is characteristic
• Macroglossia and Raccoon eye signs are pathognomonic signs

Correct Answer. a

(14). In a patient being investigated for aplastic anemia, diminished expression of which of the following markers will help in confirmation?

a. CD 19

b. CD 33

c. CD 45

d. CD 34

Solution. D
Reference: Harrison’s Principles of Internal Medicine, 20th edition
Bone marrow failure results from severe damage to the hematopoietic cell compartment. In aplastic anemia, replacement of the bone
marrow by fat is apparent in the morphology of the biopsy specimen and magnetic resonance imaging (MRI) of the spine. Cells bearing
the CD34 antigen, a marker of early hematopoietic cells, are greatly diminished, and in functional studies, committed and primitive
progenitor cells are virtually absent; in vitro assays have suggested that the stem cell pool is reduced to ≤1% of normal in severe disease
at the time of presentation.

Correct Answer. d

(15). Factor V Leiden mutation is

a. Arginine changes to Glutamine at 506th position

b. Glutamine changes to Arginine at 506th position

c. Arginine changes to Valine at 506th position

d. Valine changes to Glutamine at 506th position

Solution. A
th
Reference: Consultative Hemostasis and Thrombosis, 4 edition (2019)
Factor V Leiden results from a single nucleotide polymorphism in the factor V gene that changes arginine 506 to glutamine. Arginine 506
is the normal initial cleavage site in factor V for activated protein C, and a mutation at this site leads to slower inactivation of factor Va
and thus prolonged thrombin generation.

Correct Answer. a

(16). Prolonged prothrombin time can be seen in all except

a. Factor VII deficiency

b. Hemophilia A

c. Disseminated intravascular coagulation

d. Factor II deficiency

Solution. B
Prothrombin time is prolonged in disorders due to deficiencies of or inhibitors of coagulation factors of extrinsic and common pathway.
Haemophilia A is due to Factor VIII deficiency which is a part of intrinsic pathway.

Correct Answer. b

(17). Most commonly involved vessel by atherosclerosis is

Copyright © 2014 Delhi Academy of Medical Sciences, All Rights Reserved. 5/55
a. Abdominal aorta – lower

b. Coronary arteries

c. Popliteal arteries

d. Internal carotid arteries

Solution. A
In descending order, the most extensively involved vessels are the lower abdominal aorta, the coronary arteries, the popliteal arteries, the
internal carotid arteries, and the vessels of the circle of Willis. In humans, the abdominal aorta is typically involved to a much greater
degree than the thoracic aorta. Vessels of the upper extremities are usually spared, as are the mesenteric and renal arteries, except at
their ostia. Although most individuals tend to have a consistent degree of atherosclerotic burden in the affected vasculature, severity of
disease in one arterial distribution does not always predict its severity in another.

Correct Answer. a

(18). PAS stained section from a kidney shows the following. What is the possible gross appearance of the kidney?

a. Granular contracted

b. Flea bitten

c. Waxy

d. Enlarged

Solution. B
This lesion occurs in
severe hypertension; vessels exhibit concentric, laminated (“onion-skin”) thickening of the walls with luminal narrowing. The laminations
consist of smooth muscle cells with thickened, reduplicated basement membrane; in malignant hypertension, they are accompanied by
fibrinoid deposits and vessel wall necrosis (necrotizing arteriolitis), particularly in the kidney, giving rise to petechial hemorrhages on the
surface (flea bitten appearance).

Correct Answer. b

(19). The most common site and type of metaplasia respectively are

a. Esophagus; columnar metaplasia

b. Esophagus; squamous metaplasia

c. Respiratory tract; columnar metaplasia

d. Respiratory tract; squamous metaplasia

Solution. D
Reference: Robbins and Cotran Pathological Basis of Disease 9th edition and Basic Pathology 10th edition.
The most common epithelial metaplasia is columnar to squamous as occurs in the respiratory tract in response to chronic irritation.

Correct Answer. d

(20). All of the following are features of reversible injury except

a. Chromatin clumping

b. Detachment of ribosomes

c. Increased cellular enzymes in blood

d. Fatty change

Solution. C>A
Reference: Robbins and Cotran Pathological Basis of Disease 9th edition and Basic Pathology 10th edition. Reversible cell injury is
characterized under the light microscope by cellular swelling and fatty change, with the former being the first manifestation of almost all
forms of injuries to the cell.Due to dilation of ER, detachment of polysomes may occur. Loss of microvilli, blebbing and swelling of
mitochondria is also seen. As the cell becomes irreversibly injured, mitochondrial amorphous densities accumulate and there is
discontinuation in plasma and organelle membranes in necrosis. One of the classical nuclear changes is pyknosis characterized by nuclear
shrinkage and increased basophilia due to chromatin condensation.

Correct Answer. c

Copyright © 2014 Delhi Academy of Medical Sciences, All Rights Reserved. 6/55
(21). A 45-year old sustained severe crush injury in his limbs during a road traffic accident. Clinical examination showed that the limbs were
swollen and tense. Blood findings showed elevated creatinine kinase enzyme, a compartmental syndrome was suspected and an
emergency fasciotomy was done. During follow up the surgeon finds that the creatinine kinase levels have risen as compared to before
the procedure. Which of the following best explains the elevated creatinine kinase

a. Free radical induced damage to muscle

b. Increased circulation flushing out the residual enzyme

c. Progression from reversible injury to irreversible

d. Progression from irreversible injury to necrosis

Solution. B
Reference: Robbins and Cotran Pathological Basis of Disease 9th edition and Basic Pathology 10th edition.
Restoration of blood flow to ischemic tissues can promote recovery of cells if they are reversibly injured, but can also paradoxically
exacerbate the injury and cause cell death.As a consequence, reperfused tissues may sustainloss of cells in addition to the cells that are
irreversiblydamaged at the end of ischemia. This process, calledischemia-reperfusion injury, is clinically important because itcontributes
to tissue damage during myocardial and cerebralinfarction and following therapies to restore bloodflow.

Correct Answer. b

(22). Iron deficiency may result in misdiagnosis of

a. Sickle cell trait

b. Sickle cell disease

c. Beta thalassemia trait

d. Beta thalassemia major

Solution. C
Iron deficiency and beta thalassemia minor are both microcytic hypochromic anemias to be differentiated on HPLC and iron studies .

Correct Answer. c

(23). An adult with sickle cell disease exhibits target cells and Howell Jolly bodies in his peripheral smear. The most likely explanation for these
findings is

a. Significant hemolysis

b. Autosplenectomy

c. Chronic hepatitis B infection

d. Folate deficiency

Solution. B
Individuals with sickle cell disease undergo autosplenectomy during adolescent years. Thus when they are adults their peripheral smears
could show target cells, acanthocytes, Howell Jolly bodies.

Correct Answer. b

(24). What are the components of partial thromboplastin time in addition to phospholipid and calcium?

a. Tissue factor

b. Prothrombin

c. Thrombin

d. Silica

Solution. D
To activate the intrinsic pathway, silica beads are used and hence aPTT is tested

Correct Answer. d

Copyright © 2014 Delhi Academy of Medical Sciences, All Rights Reserved. 7/55
(25). A 35-year-old woman presents with a 5-day history of a painful sore on her back. Physical examination reveals a 1-cm abscess over her left
shoulder. Biopsy of the lesion shows vasodilation and leukocyte margination (shown in the image).What glycoprotein mediates initial
tethering of segmented neutrophils to endothelial cells in this skin lesion?

a. Cadherin

b. Integrin

c. Laminin

d. Selectin

Solution. D
• Selectins are sugar-binding glycoproteins that mediate the initial adhesion of leukocytes to endothelial cells at sites of inflammation.
• E-selectins are found on endothelial cells, P-selectins are found on platelets, and L-selectins are found on leukocytes.
• E-selectins are stored in Weibel-Palade bodies of resting endothelial cells.
• Upon activation, E-selectins are redistributed along the luminal surface of the endothelial cells, where they mediate the initial adhesion
(tethering) and rolling of leukocytes.
• After leukocytes have come to a rest, integrins mediate transendothelial cell migration and chemotaxis.
• Cadherins mediate cell-cell adhesion, but they are not involved in neutrophil adhesion to vascular endothelium.
• Entactin and laminin are basement membrane proteins.

Correct Answer. d

(26). In drug induced SLE, HLA DR6 allele is associated with

a. Procainamide

b. Hydralazine

c. Isoniazid

d. Penicillamine

Solution. A
A lupus erythematosus-like syndrome may develop in patients receiving a variety of drugs, including hydralazine, procainamide, isoniazid,
and D-penicillamine, to name only a few. Somewhat surprisingly, anti-TNF therapy, which is effective in rheumatoid arthritis and other
autoimmune diseases, can also cause drug-induced lupus. Many of these drugs are associated with the development of ANAs, but most
patients do not have symptoms of lupus erythematosus. For example, 80% of patients receiving procainamide test positive for ANAs, but
only one third of these manifest clinical symptoms, such as arthralgias, fever, and serositis. Although multiple organs are affected, renal
and central nervous system involvement is distinctly uncommon. There are serologic and genetic differences from classic SLE, as well.
Antibodies specific for double-stranded DNA are rare, but there is an extremely high frequency of antibodies specific for histones.
Persons with the HLA-DR4 allele are at a greater risk of developing a lupus erythematosus-like syndrome after administration of
hydralazine, whereas those with HLA-DR6 (but not DR4) are at high risk with procainamide. The disease remits after withdrawal of the
offending drug

Correct Answer. a

(27). The following gross photograph is from a young patient who died a sudden death. The patient also had disproportionately long limbs, lens
subluxation and a cardiac abnormality as shown in the photograph. Which gene mutation could be responsible for this condition?

a. Titin

b. Beta myosin heavy chain

c. Fibrillin 1

d. Actin

Solution. C
Marfan syndrome is caused by mutations in the FBN1 gene on chromosome 15which encodes fibrillin-1, a glycoprotein component of the
extracellular matrix. Fibrillin-1 is essential for the proper formation of the extracellular matrix, including the biogenesis and maintenance
of elastic fibers.

Correct Answer. c

(28). A syndrome consisting of osteomas, epidermoid cysts, desmoidtumors and adenomatous polyps is suggestive of

a. Gardner syndrome

Copyright © 2014 Delhi Academy of Medical Sciences, All Rights Reserved. 8/55
b. Lynch syndrome

c. Turcot syndrome

d. Cowden syndrome

Solution. A

Correct Answer. a

(29). All are standard risk karyotype findings in plasma cell myeloma except

a. t(6;14)

b. t(11;14)

c. Hyperploidy

d. t(14;16)

Solution. D
Reference: WHO Classification of Tumors of Hematopoetic and Lymphoid tissues (2017)

Correct Answer. d

(30). Which of the following type of Acute myeloid leukemia is associated with normal karyotype finding and good prognosis?

a. AML with RUNX1- RUNX1T1 fusion

b. AML with PML-RARA fusion

c. AML with BCR- ABL1 fusion

d. AML with biallelic CEBPA mutations

Solution. D
Reference: WHO Classification of Tumors of Hematopoetic and Lymphoid tissues (2017)
Normal karyotype is associated with mutations. Other options include fusions which occur due to rearrangements and can be readily
identified on karyotyping.

Correct Answer. d

(31). Most common genetic change in alpha thalassemia is

a. Deletions

b. Missense mutations

c. Nonsense mutations

d. Translocations

Solution. A
Reference: Reference: Wintrobe’s Clinical Hematology, 14th edition (2018)
α-Thalassemia is most frequently caused by deletional mutations that involve one or both α-globin genes

Correct Answer. a

(32). The following peripheral smear finding in an elderly patient with cytopenia should raise the suspicion of

a. Sepsis

b. Chronic myeloid leukemia

c. Myelodysplastic syndrome

d. Chronic neutrophilic leukemia

Copyright © 2014 Delhi Academy of Medical Sciences, All Rights Reserved. 9/55
Solution. C
The peripheral smear shows Pseudo PelgerHuet anomaly- bilobed neutrophils with reduced granules, suggestive of dysplasia in the
granulocytic lineage

Correct Answer. c

(33). Bone marrow aspirate of an 8 year old boy with an aggressively growing jaw lesion is shown below. What possible cytogenetic
abnormality can be expected on karyotyping

a. t(8;21)

b. t(9;22)

c. t(15;17)

d. t(8;22)

Solution. D
The atypical cells in the marrow show vacuolation suggestive of Burkitt lymphoma. BL classically involves translocations of c-myc gene on
chromosome 8.

Correct Answer. d

(34). Most common site of metastasis in lung carcinoma is

a. Adrenal

b. Brain

c. Liver

d. Bone

Solution. (b) Brain


th th
Reference: Robbins and Cotran’s Pathological Basis of Disease, 9 edition and Basic Pathology, 10 edition; Harrisons’ Principles of
th
Internal Medicine, 19 edition
Most common site of metastasis in lung carcinoma is brain. Most specific site of metastasis in lung carcinoma is adrenal glands.

Correct Answer. b

(35). All of the following are/have receptor tyrosine kinases except

a. EGFR

b. C-kit

c. ALK

d. JAK2

Solution. D
JAK2 and ABL1 are two classic examples of non receptor Tyrosine kinases

Correct Answer. d

(36). Oncocytes are seen in all of the following except

a. Pituitary

b. Thyroid

c. Pancreas

d. Thymus

Copyright © 2014 Delhi Academy of Medical Sciences, All Rights Reserved. 10/55
Solution. (d) Thymus
Sol:
- Oncocytes are cells with marked cytoplasmic eosinophilia and granularity.
- There is a great increase in the number of mitochondria which is typical of oncocytes morphology.
- Oncocytes are present in the acini and ducts of normal salivary gland.
They are also present in some benign tumors of salivary glands, thyroid, parathyroid, kidney, lung and pituitary.

Correct Answer. d

(37). Durck granulomas are seen in

a. Cat scratch disease

b. Leprosy

c. Cerebral malaria

d. Q fever

Solution. C
Accumulation of glial cells surrounding hemorrhagic foci in white matter is called Durck’s granuloma

Correct Answer. c

(38). Deficiency of gpIIb/IIIA causes

a. Bernard Soulier syndrome

b. Glanzmann thrombasthenia

c. Wiskott Aldrich disease

d. Disseminated intravascular coagulation

Solution. B

Correct Answer. b

(39). Following is the histopathological picture of an intra abdominal mass in a child with abdominal pain and hematuria, which crosses the
midline. On gross examination, the mass appears to be arising from the kidney and is homogenous, tan grey and soft. The microscopic
picture conveys an important finding for prognosis. Which of the following is not true regarding the prognosis?

a. Absence of p53 mutations

b. Resistance to chemotherapy

c. Increased risk of recurrence

d. Increased mortality

Solution. (a) Absence of p53 mutations


th th
Reference: Robbins and Cotran’s Pathological Basis of Disease, 9 edition and Basic Pathology, 10 edition
Approximately 5% of tumors reveal anaplasia, defined as the presence of cells with large, hyperchromatic, pleomorphic nuclei and
abnormal mitoses. The presence of anaplasia correlates with the presence of TP53 mutations and the emergence of resistance to
chemotherapy. Recall that p53 elicits pro-apoptotic signals in response to DNA damage. The loss of p53 function might explain the
relative unresponsiveness of anaplastic cells to cytotoxic chemotherapy.

Correct Answer. a

(40). Which of the following HHV8 associated tumors occurs in immunocompetent individuals?

a. Kaposi sarcoma

b. Castleman disease

c. Diffuse large B celllymphoma, NOS

d. Germinotropiclymphoproliferativedisorders

Copyright © 2014 Delhi Academy of Medical Sciences, All Rights Reserved. 11/55
Solution. D
In addition to causing Kaposi sarcoma, whichmayinvolve the lymphnodes, the humanherpesvirus HHV8 (alsocalled Kaposi sarcoma-
associatedherpesvirus) isresponsible for a spectrum of lymphoproliferativedisorders. Theseinclude HHV8positive
multicentricCastlemandisease (MCD); HHV8-positive diffuse large Bcelllymphoma (DLBCL), NOS, whichfrequently arises in the
background of MCD; and germinotropiclymphoproliferativedisorder (GLPD). Except for GLPD,these disorders are most commonly seen in
the setting of HIV infection and in HHV8endemic areas, but they can also occur in other immunosuppressed states, including following
transplantation. GLPD is most commonly seen in immunocompetent individuals. Primary effusion lymphoma (PEL) and extracavitary PEL
are also caused by HHV8.

Correct Answer. d

(41). Which of the following are seen in acute hepatitis B?

a. Ductular reaction

b. Ground glass hepatocytes

c. Spotty necrosis

d. Interface hepatitis

Solution. C

Correct Answer. c

(42). The most likely cause of the pathologic findings in the spleen shown in Fig is which of the following?

a. Amyloidosis

b. Metastatic carcinoma

c. Septic infarct

d. Hodgkin disease

Solution. (a) Amyloidosis


Ref: Read the text below
Sol:
• Amyloidosis is caused by the deposition of an abnormal proteinaceous material between cells.
• The majority of the cases are idiopathic, but a small percentage is secondary to chronic infection or inflammation, plasma cell
dyscrasias, or immune diseases. One of the characteristic presentations of amyloidosis is splenic infiltration and splenomegaly caused by
deposition of amyloid in the follicular regions.
• Grossly, the spleen has a diffuse, pink, glassy, waxy appearance with obliteration of the white pulp.
• Amyloid infiltration can also affect the kidneys, liver, and heart. Clinical symptoms are usually due to functional impairment of the
diseased organ.
• The diagnosis of amyloidosis is made by tissue biopsy or, more recently, by fat-pad biopsy looking for amyloid deposits. With Congo red
stain, amyloid appears red; with polarization, it shows an apple-green birefringence, which is diagnostic of amyloid.

Correct Answer. a

(43). The most common site of extra medullary haematopoiesis in primary myelofibrosis is

a. Lymph nodes

b. Liver

c. Spleen

d. Skin

Solution. C
Reference: WHO Classification of Tumors of Hematopoetic and Lymphoid tissues (2017) The most common site of extramedullary
haematopoiesis is the spleen, followed by the liver. The spleen shows an expansion of the red pulp by erythroid, granulocytic, and
megakaryocytic cells.

Correct Answer. c

Copyright © 2014 Delhi Academy of Medical Sciences, All Rights Reserved. 12/55
(44). An elderly patient presents with anemia for work up. Peripheral blood shows 5% blasts and bone marrow examination shows 18% blasts
with Auer rods. 6% ring sideroblasts are also seen.Hb is reduced and low platelets are seen. Monosomy 7 is identified on karyotyping. The
probable diagnosis is?

a. Acute myeloid leukemia

b. Myelodysplastic syndrome with excess blasts - 1

c. Myelodysplastic syndrome with ring sideroblasts

d. Myelodysplastic syndrome with excess blasts – 2

Solution. D
Reference: WHO Classification of Tumors of Hematopoetic and Lymphoid tissues (2017)

Correct Answer. d

(45). Special stain done on a lung biopsy from a patient with history of interstitial lung disease shows the following. What is your diagnosis?

a. Silicosis

b. Asbestosis

c. Coal worker’s pneumoconiosis

d. Berylliosis

Solution. B
The cause of interstitial lung disease is apparent here as asbestosis. The inhaled long, thin object known as an asbestos fiber becomes
coated with iron and calcium, then is called a ferruginous body, several of which are seen here with a Prussian blue iron stain. Ingestion
of these fibers by macrophages sets off a fibrogenic response through release of cytokine growth factors that promote continued collagen
deposition by fibroblasts. Some houses, business locations, and ships still contain construction materials with asbestos, particularly
insulation, so care must be taken to prevent inhalation of asbestos fibers during remodeling or reconstruction.

Correct Answer. b

(46). Which of the following is an autosomal dominant inherited cancer syndrome?

a. Fanconianemia

b. Bloom syndrome

c. Gorlin syndrome

d. Xerodermapigmentosum

Solution. C

Correct Answer. c

(47). Histopathology of mitral valve prolapse shows what kind of change?

a. Myxomatous

b. Fibrinoid

c. Granulomatous

d. Fibrous

Copyright © 2014 Delhi Academy of Medical Sciences, All Rights Reserved. 13/55
Solution. (a) Myxomatous
Ref.:Read the text below
Sol :
Pathology of valves in mitral valve prolapsed:
- The characteristic anatomic change in MVP is interchordalballoonin g (hooding) of the mitral leaflets or portions thereof the affected
leaflets are ofte enlarged, redundant, thick, and rubbery. The associated tendinous cords may be elongated, thinned, or even ruptured
and the annulus may e dilated.
Histologically :
There is attenuation of the collagenous fibrosa layer of the valve, on which the structural integrity of the leaflet depends, accompanied by
marked thickening of the spongiosa layer with deposition of mucoid (myxomatous) material.

Correct Answer. a

(48). Following is not true concerning amyloidosis:-

a. Amyloidosis associated with multiple myeloma has the poorest prognosis.

b. Fine - needle biopsy of subcutaneous abdominal fat is a simple & reliable method for diagnosing secondary systemic amyloidosis.

c. Hepatic amyloid disease produces hepatomegaly but rarely jaundice.

d. Amyloidosis of the spleen is associated with severe anemia.

Solution. (d) Amyloidosis of the spleen is associated with severe anemia.


Ref:Read the text below
Sol:
• Amyloidosis of the spleen characteristically is not associated with leukopenia & anemia.
• When Amyloidosis develops in patients with multiple myeloma, manifestations leading to initial hospitalization are more apt to be
related to amyloid disease than to myeloma.
• In these cases prognosis is very poor, and life expectancy is usually less than 6 months.

Correct Answer. d

(49). Bone deformities of Cooley’s anemiainclude

a. Hair on end appearance

b. Prominent zygomatic bones

c. Rib within rib appearance

d. All of the above

Solution. D
Reference: Wintrobe’s Clinical Hematology, 14th edition (2018)
Inadequate red cell transfusions lead to the development of typical bone abnormalities, which were described in the first reports of the
disease and are caused by increased erythropoiesis, and consequent expansion of the bone marrow (up to 15-30 times normal). The skull
is large and deformed by frontaland posterior bossing with the diploe increased in thickness. The outer and inner The zygomatic bones
aretables are thin and the trabeculae arranged in vertical striations, resulting in a “hair-on-end” appearance on x-ray. The zygomatic
bones are prominent, the base of the nose is depressed, and pneumatization of the sinuses is delayed. Overgrowth of the maxilla produces
severe malocclusion, with a rodent-like appearance. Metatarsal andmetacarpal bones expand as a consequence of increased
erythropoiesis. The ribs are broad, often with a “rib-within-rib” appearance, and the vertebral bodies are square. The trabeculation of the
medullary space gives the bones a mosaic pattern. Shortening of long bones is common, resulting from premature fusion of the humeral
and femoral epiphyseal lines.Extramedullary erythropoiesis gives riseto masses that protrude from bones where red marrow persists.

Correct Answer. d

(50). An asymptomatic patient with incidentally detected mild anemia and no history of blood transfusion is being worked up. MCV is slightly
less, RDW is normal, and HPLC showed raised HbA2 (6%) with almost normal HbA. The provisional diagnosis is

a. Beta thalassemia major

b. Iron deficiency anemia

c. Beta thalassemia minor

d. Beta thalassemia intermedia

Solution. C
th
Reference: Wintrobe’s Clinical Hematology, 14 edition (2018)

Copyright © 2014 Delhi Academy of Medical Sciences, All Rights Reserved. 14/55
Correct Answer. c

(51). A patient with history of smoking presents with weight loss, hemoptysis, cough and dyspnea. A sputum cytological examination was
performed and the following finding was obtained. Which of the following is the most probable diagnosis?

a. Adenocarcinoma

b. Squamouscell carcinoma

c. Neuroendocrine tumor

d. Large cell carcinoma

Solution. B
The photograph shows orange-staining, keratinized squamous carcinoma cell with a prominent hyperchromatic nucleus (large arrow).
Note the size of the tumor cells compared with normal neutrophils (small arrow).

Correct Answer. b

(52). The following histopathology shows a pathological finding in the kidneys of a patient with hypertension. Which of the options below is
most apt for the expected gross finding?

a. Granular kidney

b. Flea bitten kidney

c. Waxy kidney

d. Scarred kidney

Solution. A
The photograph shows a igh-power view of two arterioles with hyaline deposition, marked thickening of the walls, and a narrowed lumen.
The kidneys are either normal or moderately reduced in size, with average weights between 110 and 130 gm. The cortical surfaces have a
fine, even granularity that resembles grain leather. The loss of mass is due mainly to cortical scarring and shrinking.

Correct Answer. a

(53). Which of the following is not involved in polyarteritisnodosa?

a. Pulmonary arteries

b. Bronchial arteries

c. Renal arteries

d. Mesenteric arteries

Solution. (a) Pulmonary arteries


Reference: Harrisons’ Principles of Internal Medicine, 19th edition
PAN is a multisystem, necrotizing vasculitis of small and medium-sized muscular arteries in which involvement of the renal and visceral
arteries is characteristic. Polyarteritisnodosa does not involve pulmonary arteries, although bronchial vessels may be involved;
granulomas,significant eosinophilia, and an allergic diathesis are not observed.

Correct Answer. a

(54). Concerning nephroblastoma, all of the following statements are true except :

a. About 10 percent of cases are bilateral

b. It originates from cells of the metanephrogenicblastema

c. It is a golden yellow mass with focal hemorrhage and necrosis

d. Some patients have a deletion in a short arm of chromosome 11

Copyright © 2014 Delhi Academy of Medical Sciences, All Rights Reserved. 15/55
Solution. (c) It is a golden yellow mass with focal hemorrhage and necrosis
Ref: Read the text below.
Sol:
- Nephroblastoma (Wilms’ tumor) is a congenital malignant neoplasm. It makes up about 5 percent of renal cancer. Boys and girls are
equally effected. The average age at which it is detected is 2 to 3 years.
- This neoplasm is significantly associated with birth defects such as aniridia and hemihypertrophy.
- Children with Wilms’ tumor and aniridia often have deletion of a segment of a short arm of chromosome 11.
- Nephroblastoma originates from cells of the metanephrogenicblastema.
Grossly the nephroblastoma is grayish white, soft and brain-like. It begins in the renal cortex and eventually replaces almost the entire
kidney.
- Microscopically there are sheets of closely packed, atypical spindle cells within which there are embedded well-formed tubular or
rosette-like structures.
- Glomeruloid structures are present. Smooth or striated muscle or cartilage may be present.
- In about 10 percent of cases, the neoplasm occurs bilaterally.
Metastases are common in the lungs, liver, brain, ands regional lymph nodes.

Correct Answer. c

(55). Concentric “onion skin” thickening and fibrosis is a feature of:

a. Primary sclerosing cholangitis

b. Wegener granulomatosis

c. Benign hypertension

d. Diabetes mellitus

Solution. (a)Primary sclerosing cholangitis


Ref:Read the text below
Sol:
• Concentric “onion skin” thickening and fibrosis may be seen most commonly in malignant hypertension but also in other conditions such
as primary sclerosing cholangitis.

Correct Answer. a

(56). The most important cause of hemolysis in alpha thalassemia is

a. Excess beta chains

b. Reduced alpha chains

c. Excess alpha chains

d. Reduced beta chains

Solution. A
Reference: Reference: Wintrobe’s Clinical Hematology, 14th edition (2018)
The primary defect in α-thalassemia is the imbalance of globin biosynthesis, which leads to an excess of β- and/or γ-globin chains. Unlike
α-chains, which are highly unstable and unable to form soluble tetramers, excess γ-chains in fetal life and β-chains in extrauterine life
associate to form relatively soluble γ4 tetramers (Hb Bart) and β4 tetramers (HbH), respectively. These excess non-α-chains damage
mature RBCs and, to a lesser extent, erythroid precursors, leading to hemolysis and, to a lesser degree, ineffective erythropoiesis.

Correct Answer. a

(57). The following finding on peripheral smear of a patient with megaloblastic anemia contains

a. DNA remnants

b. Ribosomes

c. Precipitated Hemoglobin

d. Microtubules

Solution. D
The finding is that of Cabot rings, which are curvilinear erythrocyte inclusions, which form a closed circle or oval, sometimes in a figure of
eight formation. Cabot rings and the granules that accompany them are the remnants of the mitotic spindle. They are seen after
splenectomy, in megaloblastic anaemia, in acute haemolytic anaemia, in the extramedullaryhaemopoiesis of primary myelofibrosis and in
myelodysplasia with marked dyserythropoiesis.

Copyright © 2014 Delhi Academy of Medical Sciences, All Rights Reserved. 16/55
Correct Answer. d

(58). All are true about anemia of chronic disease except

a. TIBC is low

b. IL-6 is high

c. EPO is high

d. Hepcidin is high

Solution. C
Anemia of chronic disease is an anemia of persistent systemic inflammation and is associated with low serum iron, reduced total iron-
binding capacity, and abundant stored iron in tissue macrophages. Several effects of inflammation contribute to the observed
abnormalities. Most notably, certain inflammatory mediators, particularly interleukin-6 (IL-6), stimulate an increase in the hepatic
production of hepcidin. Hepcidin inhibits ferroportin function in macrophages and reduces the transfer of iron from the storage pool to
developing erythroid precursors in the bone marrow. As a result, the erythroid precursors are starved for iron in the midst of plenty. In
addition, these progenitors do not proliferate adequately because erythropoietin levels are inappropriately low for the degree of anemia.

Correct Answer. c

(59). Following is a histopathological picture from an abdominal mass. The finding is suggestive of

a. Hyperplasia

b. Dysplasia

c. Metaplasia

d. Anaplasia

Solution. D
Marked variation in cell and nuclear sizes, the hyperchromatic nuclei, and the presence of tumor giant cells is suggestive of malignancy
and hence the answer is anaplasia.

Correct Answer. d

(60). The immunophenotype CD 19 positive, CD 79a positive, HLA DR positive, TdT positive, CD 3 negative, MPO negative best correlates with
which of the following cells

a. Cytotoxic T cell

b. Plasma cell

c. B Lymphoblast

d. Myeloblast

Solution. -NA-

Correct Answer. c

(61). All of the following are false about inflammatory myofibroblastictumor except

a. Male preponderance

b. ALK rearrangements are seen

c. Neutrophil rich

d. Mostly multiple

Copyright © 2014 Delhi Academy of Medical Sciences, All Rights Reserved. 17/55
Solution. B
Inflammatory myofibroblastictumor
• More common in children
• Equal male-to-female ratio
• Imaging studies show a single (rarely multiple) round, well-defined, usually peripheral mass with calcium deposits in about a quarter of
cases
• Grossly, the lesion is firm, 3 to 10 cm in diameter, and grayish white
• Microscopically, there is proliferation of spindle-shaped fibroblasts and myofibroblasts, lymphocytes, plasma cells, and peripheral
fibrosis
• Some of these tumors have activating rearrangements of the anaplastic lymphoma kinase (ALK) gene, located on 2p23
• Treatment with ALK kinase inhibitors have produced sustained responses in such cases.

Correct Answer. b

(62). Sipple Syndrome is characterised by

a. Bilateral pheochromocytomas

b. RET protoncogene mutations

c. Parathyroid hyperplasia

d. All of the above

Solution. D
MEN-2A, or Sipple syndrome, is characterized by pheochromocytoma,
medullary carcinoma of the thyroid, and parathyroid hyperplasia
• Medullary carcinomas of the thyroid occur in almost 100% of patients
• They are usually multifocal and are virtually always associated with foci of C-cell hyperplasia in the adjacent thyroid
• The medullary carcinomas may elaborate calcitonin and other active products and are usually clinically aggressive
• Among individuals with MEN-2A, 40% to 50% have pheochromocytomas, which are often bilateral and may arise in extra-adrenal sites
• Parathyroid hyperplasia and evidence of hypercalcemia or renal stones occur in 10% to 20% of patients
• MEN-2A is clinically and genetically distinct from MEN-1 and is caused by germline gain-of-function mutations in the RET proto-
oncogene on chromosome 10q11.2.

Correct Answer. d

(63). Primary granules of neutrophils characteristically contain

a. Collagenase

b. Myeloperoxidase

c. Lactoferrin

d. Alkaline phosphatase

Solution. B
Reference: Wintrobe’s Clinical Hematology, 14th edition (2018)
The azurophilic, or primary, granules are formed during the promyelocytic stage and in general contain many antimicrobial compounds.
These granules fuse with phagocytic vesicles, resulting in the delivery of their contents to the ingested organism. Among the azurophilic
granule contents is myeloperoxidase (MPO), a protein that catalyzes the production of hypochlorite (OCl−) from chloride andhydrogen
peroxide produced by the oxidative burst. It makes up 5% of the dry weight of neutrophil and imparts green color to pus.

Correct Answer. b

(64). The following peripheral smear picture can be seen in

a. Beta thalassemia

b. Hereditary spherocytosis

c. Post splenectomy

d. Both A and C

Solution. D
The peripheral smear shows target cells and a Howell Jolly body. This picture can be seen in both A and C

Correct Answer. d

Copyright © 2014 Delhi Academy of Medical Sciences, All Rights Reserved. 18/55
(65). A patient presented with hematuria, flank pain and abdominal mass. On CT, a solid mass with necrosis is seen in the kidney. Post
operative gross specimen is yellowish on cut section and shows the following histopathological picture. Which of the following is true
about this condition?

a. Arisesfrom proximal tubular cell

b. MET proto oncogene involvement

c. Least common subtype

d. Seen in sickle cell anemia

Solution. A
The history is suggestive of renal cell carcinoma. Gross appearance is suggestive of clear cell carcinoma as it contains lipid inclusions
giving a yellow appearance to the lesion. The histopathology shows nests of tumor cells with clear cytoplasm. The diagnosis is clear cell
carcinoma. This is the most common subtype of renal cell carcinoma and arises from proximal tubular cells. MET proto oncogene
involvement is seen in papillary RCC. And medullary subtype is seen in sickle cell trait patients.

Correct Answer. a

(66). All are true about paroxysmal nocturnal hemoglobinuria except

a. Only hemolyticanemia with acquired genetic defect

b. MIRL is the most important protein affected

c. Not considered to be a pre leukemic condition

d. Gold standard for diagnosis is flow cytometry

Solution. C
About 5% to 10% of patients eventually develop acute myeloid leukemia or a myelodysplastic syndrome, possibly because hematopoietic
stem cells have suffered some type of genetic damage. Rest of the statements are true.

Correct Answer. c

(67). Most common cause of myelophthisicanemia is

a. Metastatic cancer

b. Storage disorders

c. Radiotherapy

d. Granulomatous disorders

Solution. A
Myelophthisicanemia describes a form of marrow failure in which space-occupying lesions replace normal marrow elements. The
commonest cause is metastatic cancer, most often carcinomas arising in the breast, lung, and prostate. However, any infiltrative process
(e.g., granulomatous disease) involving the marrow can produce identical findings. Myelophthisicanemia is also a feature of the spent
phase of myeloproliferative disorders.

Correct Answer. a

(68). Types of bronchial carcinoma seen in non smokersis:-

a. Squamous cell carcinoma (SCC)

b. Adenocarcinoma

c. Small cell carcinoma (oat cell carcinoma)

d. Alveolar cell or bronchoalveolar carcinoma

Copyright © 2014 Delhi Academy of Medical Sciences, All Rights Reserved. 19/55
Solution. (b) Adenocarcinoma
Ref: Read the text below
Sol:
Types of bronchial carcinoma:
• Squamous cell carcinoma (SCC)
• Adenocarcinoma
• Small cell carcinoma (oat cell carcinoma)
• Alveolar cell or bronchoalveolar carcinoma
Squamous cell carcinoma:
• 60% of all lung tumours
• Associated with smoking and is rare in non-smokers
• Squamous metaplasia -> carcinoma in situ -> invasive carcinoma
Adenocarcinoma:
• 15% of lung tumours
• Has a tendency to be more peripheral, arising in the small bronchial glands
• Most common in women
• Is the type seen in non-smokers
Small cell (oat cell) carcinoma:
• 20% of lung tumours
• Arises from the Kulchitskychromaffin cells
• Highly malignant
• Hormone production by the tumour is common
• A benign form of a small cell carcinoma is a carcinoid tumour

Correct Answer. b

(69). The cell labeled A is best described as which of the following?

a. Myeloblast

b. Proerythroblast

c. Metamyelocyte

d. Myelocyte

Solution. (a) Myeloblast


Ref: Read the text below
Sol:
• The first stage in granulopoiesis is the myeloblast (A), a large cell with prominent light- staining nucleoli with only a little cytoplasm,
generally without granules.
• The lineage shown in the figure illustrates eosinophilic development in the bone marrow. Basophils may be bilobed or segmented, but
with larger and more irregular granules that obscure the nucleus in Wright-stained blood smears.
• The promyelocyte (B) is the next cell in the lineage. It is larger than the myeloblast, nucleoli are less visible, and primary granules are
present in the cytoplasm. Granule specificity is attained in the myelocyte with flattening of the nucleus. The eosinophilic myelocyte (C)
differentiates into the eosinophilic metamyelocyte (D) when invagination of the nucleus begins. Further invagination leads to the
formation of an eosinophilic band (E) and ultimately a mature eosinophil (F). An eosinophil has a bilobed nucleus and plays an important
role in allergic and parasitic infections.
• The granules stain with eosinophilic dyes and contain major basic protein, histaminase, peroxidase, and some hydrolytic enzymes.
Eosinophils have an affinity for antigenantibody complexes and, although phagocytic, are not as active against bacteria as neutrophils.
• The histaminase secreted by eosinophils counteracts the release of histamine from basophils and mast cells, essential in hypersensitivity
reactions. B lymphocytes differentiate into antibody-producing plasma cells; T lymphocytes are responsible for cell-mediated responses
including graft rejection; and neutrophils are responsible for phagocytosis of bacteria.

Correct Answer. a

(70). Which of the free radical scavenger enzyme is not correctly matched with its location in the cell

a. Super Oxide Dismutase (SOD): cytosol

b. Superoxide Dismutase (SOD): mitochondria

c. Catalase: Mitochondria

d. Glutathione peroxidase: cytosol

Solution. C
Reference: Robbins and Cotran Pathological Basis of Disease 9th edition and Basic Pathology 10th edition.
SOD is of two types- mitochondrial and cytosolic. Catalase is a peroxisomal enzyme. Glutathione peroxidase is a cytosolic enzyme.

Correct Answer. c

Copyright © 2014 Delhi Academy of Medical Sciences, All Rights Reserved. 20/55
(71). Which of the following is the most potent free radical

a. Hydroxyl an ion

b. Superoxide anion

c. Peroxy nitrite anion

d. Hydrogen peroxide

Solution. A
Reference: Robbins and Cotran Pathological Basis of Disease 9th edition and Basic Pathology 10th edition.
Hydroxyl ion is the most potent free radical and is principally responsible for damaging lipids, protein and DNA

Correct Answer. a

(72). The cells shown in the photograph are increased in which of the following infections

a. Bacterial

b. Fungal

c. Viral

d. Helminthic

Solution. D
The histopathology picture shows abundance of eosinophils, showing possibility of helminthic infections.

Correct Answer. d

(73). Which of the following is false about well differentiated liposarcoma?

a. Common in adults

b. Associated with p53 mutations

c. Amplification of MDM2 seen

d. CDK4 positivity

Solution. B
Biallelic abnormalities of the TP53 gene are found in virtually every type of cancer, including carcinomas of the lung, colon, and
breast—the three leading causes of cancer deaths. In most cases, mutations affecting both TP53 alleles are acquired in somatic cells. In
other tumors, such as certain sarcomas like liposarcoma, the TP53 gene is intact but p53 function is lost because of amplification and
overexpression of the MDM2 gene, which encodes a potent inhibitor of p53. CDK4, MDM2 and p16 are positive.

Correct Answer. b

(74). The following is a lung biopsy from a young patient with cough, evening rise of fever, malaise and night sweats. Which special stain will
help in the diagnosis?

a. Periodic acid Schiff stain

b. ZiehlNeelson stain

c. Warthin Starry stain

d. Masson Fontana stain

Solution. B
The history is suggestive of tuberculosis and the photograph shows caseating granulomas with normal lung tissue. The stain used to
demonstrate TB bacilli is ZN (acid fast) stain.

Correct Answer. b

(75). Which of the following statements about miRNAs is/are TRUE?

Copyright © 2014 Delhi Academy of Medical Sciences, All Rights Reserved. 21/55
a. Consist of RNA 19 to 24 nucleotides in length

b. Cannot be evaluated in array format as part of clinical studies

c. Alters gene expression and protein translation

d. A and C

Solution. D
MicroRNA (miRNA) consists of an RNA sequence of 19-24 nucleotides in length and participates in cell processes such as apoptosis and
development via the effect on gene expression and protein translation. Recently miRNA signatures have been evaluated In clinical
samples from patients with lung cancer, CLL and breast cancer. In a recent lung cancer study, the miRNA profile correlated with survival.

Correct Answer. d

(76). Which of the following statements is TRUE regarding telomerase?

a. It has both a DNA and a protein component.

b. It is a DNA topoisomerase.

c. Telomerase protects the integrity of the chromosomal ends.

d. Overexpression of telomerase is found in all cancer specimens

Solution. C
Telomerase is a DNA polymerase that synthesises the repeating six base pair motif (TTAGGG) that compromises the ends of all
chromosomes. It is a nucleoproteins with both a protein and RNA component. With cell division the telomere ends become progressively
shorter until a critical length is reached and programmed cell death is initiated. Synthesising and repairing the ends of the shortening
telomeres allows cells to maintain the integrity of the chromosomal ends and is important for cells that need to divide without reaching
senescence such as cancer cells. In light of this it is not surprising that the majority of cancer cells overexpress telomerase but there are
cancers that appear to have invoked alternative mechanisms to repair telomeres. Indeed telomerase deficient mice can be induced to
develop tumors.

Correct Answer. c

(77). All the following are causes of secondary amyloidosis except :

a. Multiple myeloma

b. Rheumatoid arthritis

c. Osteomyelitis

d. Bronchiectasis

Solution. (a) Multiple myeloma


Ref: Read the text below
Sol :
- Multiple myeloma is associated with primary amyloidosis.
- Primary amyloidosis is associated with immunologicaldyscrasia/B-cell neoplasm.
It is usually AL type amyloidosis.

Correct Answer. a

(78). A 23-year-old female sought medical help because of a painless asymmetrical enlargement of the lower neck. The patient had no history of
dyspnea, dysphagia, hoarseness, or previous radiation exposure. On physical examination, besides the enlarged asymmetrical thyroid
gland, there was also a palpable lymphadenopathy. A lymph node biopsy (see Figure) was performed. Hematoxilin and eosin (H&E)
stained slide shows the lesion. What is the most appropriate diagnosis?

a. Medullary carcinoma of the thyroid

b. Follicular carcinoma

c. Papillary carcinoma

d. Anaplastic carcinoma

Copyright © 2014 Delhi Academy of Medical Sciences, All Rights Reserved. 22/55
Solution. (c) Papillary carcinoma
Ref: Read the text below
Sol :
• Papillary carcinoma of the thyroid is the most common form of thyroid cancer. Most cases are seen between the second and third
decade of life and are associated with previous radiation therapy.
• Many times the first manifestation is a metastasis to the regional neck nodes. The histologic characteristics of papillary carcinoma are
branching papillae with single or multiple layers of cuboidal to columnar cells.
• The characteristic appearance of the nucleus is rather clear, ground-glass (orphan Annie) nuclei.
• Characteristic intracytoplasmic inclusions, and occasional grooves, are seen. Psammoma bodies are often present in the papillae.
• The most common variant of papillary carcinoma is the follicular variant, in which the tumor cells form follicular architecture; however,
the nuclear changes, as well as focal areas of papillary structures, are enough to make the differential diagnosis from follicular
carcinoma.

Correct Answer. c

(79). All of the following are true about synovial sarcoma except

a. Occurs only in synovium

b. Cytogenetic change is t(x;18)

c. Biphasic pattern of morphology

d. TLE1 positive

Solution. A
Synovial sarcoma was so-named because the first described cases arose in the soft tissues near the knee joint and a morphologic
relationship to synovium was postulated. However, this name is a misnomer, as these tumors can present in locations (chest wall, head
and neck) that lack synovium and their morphologic features are inconsistent with an origin from synoviocytes. Synovial sarcomas
account for approximately 10% of all soft tissue sarcomas and rank as the fourth most common sarcoma. Most occur in people in their 20s
to 40s. Patients usually present with a deep-seated mass that has been present for several years. Most synovial sarcomas show a
characteristic chromosomal translocation t(x;18)s(p11;q11) producing SS18-SSX1, -SSX2, or -SSX4 fusion genes that encode chimeric
transcription factors. Both monophasic and biphasic patterns are seen.
TLE1: 80 - 90%, relatively specific and sensitive marker.
CK, EMA, CD 99 and BCL2 are positive

Correct Answer. a

(80). A couple are expecting a child. HbA2 level of the mother is 5.6% and that of the father is 3%. They are worried about occurrence of beta
thalassemia in their child and want to know the percentage risk of both major and minor in the child, respectively. What is the risk
possible?

a. 50%, 50%

b. 0%, 50%

c. 50%, 0%

d. 0%, 0%

Solution. B
Beta thalassemia is an autosomal recessive condition. HbA2 level of 5.6% in the mother shows that she is beta thalassemia minor while
the father is normal. Hence, the risk of major in child is 0% while of minor is 50%.

Correct Answer. b

(81). Angiogenesis is stimulated by all except

a. PDGF

b. VEGF

c. FGF

d. HGF

Solution. (d) HGF


Ref.: Read the text below
Sol :

Copyright © 2014 Delhi Academy of Medical Sciences, All Rights Reserved. 23/55
Correct Answer. d

(82). Frozen section is employed for the following purposes except

a. Fat demonstration

b. Amyloid

c. Final diagnosis

d. Enzymes

Solution. C
Frozen or intraoperative evaluation is used for demonstration of substances which are usually lost during formalin fixation and
processing. Fat is one such substance. Determination of enzymes when they are labile in nature is preferred on frozen section than on
formalin fixed tissue. Rapid confirmed diagnosis is not possible, however a general differential diagnosis can be rendered on frozen
section.

Correct Answer. c

(83). In systemic senile amyloidosis the biochemical form of amyloid is

a. Aβ

b. ATTR

c. AA

d. Aβ2m

Solution. B

Correct Answer. b

(84). The DNA damage checkpoints are located in which phase of the cell cycle?

a. G1/S

b. S/G2

c. M

d. All of the above

Solution. A
There are three major checkpoints, occurring at G1/S, G2/M and S. These pathways promote cell cycle arrest and coordinate the
recruitment of repair proteins to the sites of DNA damage.

Correct Answer. a

(85). Which of the following are receptor tyrosine kinases?

a. Platelet-derived growth factor receptor

b. Insulin-like growth factor receptor 1

c. cKit

d. All of the above

Solution. D
All are proto oncogenes, and receptor tyrosine kinases.

Correct Answer. d

(86). During atrophy some of the cellular debris containing autophagic vacuole may resist proteolysis and persist in the cells, which of the
following is an example of such residual body?

Copyright © 2014 Delhi Academy of Medical Sciences, All Rights Reserved. 24/55
a. Fatty liver

b. Brown atrophy

c. Scleral icterus

d. Melanin in skin

Solution. B
th
Reference: Robbins and Cotran Pathological Basis of Disease 9th edition and Basic Pathology 10 edition.
Atrophy results from decreased protein synthesis and increased protein degradation in cells. Protein synthesis decreases because of
reduced metabolic activity. Degradation of cellular proteins occurs mainly by the ubiquitin proteasome pathway. In many cases, atrophy is
also accompanied by increased autophagy, marked by appearance of increased number of autophagic vacuoles. Some of the cellular
debris within the vacuoles may resist digestion and persist in the cells as membrane bound residual bodies, an example of which is
lipofuscin granules which when accumulate can lead to a brownish discoloration of the tissue called Brown Atrophy.

Correct Answer. b

(87). The following bone marrow aspirate picture is from a 60 year old patient with splenomegaly and raised total leucocyte count. The
karyotype done would show

a. t(8;21)

b. t(15;17)

c. t(9;22)

d. t(16;16)

Solution. C
The aspirate shows basophilia, metamyelocytes, myelocytes with raised TLC and splenomegaly suggestive of CML. Karyotype would show
Philadelphia chromosome.

Correct Answer. c

(88). The following is a gross specimen of a colectomy in a 38 year old patient, with fundic gland polyps in stomach and a family history of
cancer. Which gene will most likely be seen in this patient?

a. P53

b. MSH2

c. APC

d. KRAS

Solution. C
The diagnosis is FAP, familial adenomatous polyposis. APC gene is mutated in FAP and increases risk of colorectal cancer.

Correct Answer. c

(89). Which protein does FLIP bind to in order to evade apoptosis?

a. APAF 1

b. Cytochrome C

c. Pro caspase 8

d. Pro caspase 9

Solution. C
Extrinsic pathway of apoptosis can be inhibited by a protein called FLIP, which binds to pro- caspase-8 but cannot cleave and activate the
caspase because it lacks a protease domain. Some viruses and normal cells produce FLIP and use this inhibitor to protect themselves
from Fas- mediated apoptosis.

Correct Answer. c

Copyright © 2014 Delhi Academy of Medical Sciences, All Rights Reserved. 25/55
(90). Chronic ITP is characterised by the following features except

a. Splenomegaly or normal sized spleen may be present

b. Reduced platelet lifespan

c. Reduced number of megakaryocytes in the bone marrow

d. Demonstration of anti platelet IgG antibody

Solution. C
Page 658, Robbins and Cotran’s pathological basis of disease, 9th edition
• The principal changes of thrombocytopenic purpura are found in the spleen, bone marrow, and blood, but they are not specific.
• The spleen is of normal or increased size.
• Typically, there is congestion of the sinusoids and enlargement of the splenic follicles, often associated with prominent reactive
germinal centers.
• The marrow reveals a modestly increased number of megakaryocytes. Some are apparently immature, with large, nonlobulated, single
nuclei. These findings are not specific but merely reflect accelerated thrombopoiesis, being found in most forms of thrombocytopenia
resulting from increased platelet destruction.
• The peripheral blood often reveals abnormally large platelets (megathrombocytes), which are a sign of accelerated thrombopoiesis.
• The autoantibodies, most often directed against platelet membrane glycoproteins IIb-IIIa or Ib-IX, can be demonstrated in the plasma
and bound to the platelet surface in about 80% of patients. In the overwhelming majority of cases, the antiplatelet antibodies are of the
IgG class.

Correct Answer. c

(91). A 30-year-old woman arrives at the emergency room complaining of fatigue and dark-colored urine. While obtaining the history of her
present illness, you learn that she has been recovering from a recent bout of pneumonia, for which she had been treated appropriately by
her primary care physician with a course of antibiotics. Physical examination reveals an enlarged spleen and slight scleral icterus. You
obtain a blood sample and see the following. What is your diagnosis?

a. Anemia of chronic disease

b. Iron deficiency anemia

c. Autoimmune haemolytic anemia- cold antibody type

d. Autoimmune haemolytic anemia- warm antibody type

Solution. C
th
Reference: Page number 644, Robbins and Cotran’s Pathological Basis of Disease, 9 edition
• Cold agglutinin type of immune hemolyticanemia is caused by IgM antibodies that bind red cells avidly at low temperatures (0°C to
4°C).
• Cold agglutinin antibodies sometimes appear transiently following certain infections, such as with Mycoplasma pneumoniae,
EpsteinBarr virus, cytomegalovirus, influenza virus, and human immunodeficiency virus (HIV).
• In these settings the disorder is self-limited and the antibodies rarely induce clinically important hemolysis.
• Chronic cold agglutinin immunohemolyticanemia occurs in association with certain B-cell neoplasms or as an idiopathic condition
• Clinical symptoms result from binding of IgM to red cells in vascular beds where the temperature may fall below 30°C, such as in
exposed fingers, toes, and ears.
• IgM binding agglutinates red cells and fixes complement rapidly.
• As the blood recirculates and warms, IgM is released, usually before complement-mediated hemolysis can occur

Correct Answer. c

(92). Which of the following are risk factors for head and neck squamous cell carcinoma?

a. Tobacco use

b. Alcohol use

c. Fanconianemia

d. All of the above

Solution. D
Several risk factors for the development of HNSCC have been identified. Two most important are tobacco and alcohol use. They seem to
be synergistic. Other risk factors include occupational exposure to chemicals and irritants, such as aromatic hydrocarbons and wood dust
and viruses such as HPV and EBV. In addition, patients with Fanconianemia are also at increased risk.

Correct Answer. d

Copyright © 2014 Delhi Academy of Medical Sciences, All Rights Reserved. 26/55
(93). The first evidence of Hemoglobin on light microscopy is seen in

a. Pro erythroblast

b. Basophilic erythroblast

c. Polychromatic erythroblast

d. Orthochromatic erythroblast

Solution. C
Reference: Reference: Wintrobe’s Clinical Hematology, 14th edition (2018)
The first faint blush of Hb, as indicated by one or more pink areas near the nucleus in dry fixed preparations,introduces
thepolychromatophilic erythroblast

Correct Answer. c

(94). Dohle bodies are composed of

a. Azurophilic granules

b. Mitochondria

c. Endoplasmic reticulum

d. Nuclear fragments

Solution. C
Reference: Reference: Wintrobe’s Clinical Hematology, 14th edition (2018)
Increased granulation is usually a reaction to infection or inflammation and is therefore referred to as toxic granulation. However, it does
also occur as a normal phenomenon, during pregnancy. Cytoplasmic inclusions may be present as an inherited or acquired abnormality.
The commonest suchabnormality is a small, pale, blue‐grey inclusion that occurs both during pregnancy and in infection and inflammation
and is known as a Döhle body which are dilated ER. Another common cytoplasmic abnormality, which is strongly suggestive of infection,
is cytoplasmic vacuolation.

Correct Answer. c

(95). The following supravital stained peripheral blood smear shows a characteristic finding. The probable diagnosis is

a. Alpha thalassemia

b. Beta thalassemia

c. Glucose 6 phosphate dehydrogenase deficiency

d. Paroxysmal nocturnal hemoglobinuria

Solution. A
The picture shows golf ball inclusions, which are formed by precipitation of tetramers of the beta chain of haemoglobin, which is present
in excess over the alpha chain in haemoglobin H disease (deletion of three of the four alpha chain genes). With supravital preparations
(brilliant cresyl blue or new methylene blue), they are seen as rounded bluish inclusions distributed regularly through the cell in relatively
high numbers (from 20 to 50 per cell). The erythrocyte thus has a ‘golf-ball’ appearance.

Correct Answer. a

(96). PML- RARA gene fusion is seen in

a. Acute promyelocyticleukemia

b. Chronic myeloid leukemia

c. Chronic lymphocytic leukemia

d. Acute lymphoblastic leukemia

Solution. A
In acute promyelocyticleukemia, due to translocation t(15;17) there is fusion of PML and RARA gene leading to maturation arrest.

Correct Answer. a

Copyright © 2014 Delhi Academy of Medical Sciences, All Rights Reserved. 27/55
(97). An adult male patient from a hilly area with fever, splenomegaly and increased total leucocyte count is being investigated. Hb is 16.5
g/dL, differential count shows increased metamyelocytes and myelocytes and platelet count is normal. The next investigation should be

a. Bone marrow biopsy with reticulin stain

b. JAK2 mutation analysis

c. BCR-ABL1 fusion analysis

d. Serum EPO levels

Solution. D
Serum EPO level here will help to distinguish between benign and malignant polycythemia since the patient is from hilly area.

Correct Answer. d

(98). Transfusion related lung injury is associated with antibodies against

a. HLA class I antigens

b. HLA class II antigens

c. Neutrophil antigens

d. All of the above

Solution. D
Antibodies to HLA class I and II antigens and to neutrophil antigens have all been clearly implicated as causing TRALI. Evidence from
hemovigilance schemes and laboratories specializing in TRALI investigation has shown that the majority of cases (75–90%) are associated
with HLA antibodies, and that with improved detection techniques about 50% of these are directed against HLA class II antigens.

Correct Answer. d

(99). All of the following viruses have been implicated in causation of hematolymphoid neoplasms except

a. EBV

b. CMV

c. HHV8

d. HTLV

Solution. B
• Three lymphotropic viruses—human T-cell leukemia virus-1 (HTLV-1), Epstein-Barr virus (EBV), and Kaposi sarcoma herpesvirus/human
herpesvirus-8 (KSHV/HHV-8)—have been implicated as causative agents in particular lymphomas.
• HTLV-1 is associated with adult T-cell leukemia/ lymphoma.
• EBV is found in a subset of Burkitt lymphoma, 30% to 40% of Hodgkin lymphoma (HL), many B-cell lymphomas arising in the setting of
T-cell immunodeficiency, and rare NK-cell lymphomas.
• In addition to Kaposi sarcoma. KSHV is associated with an unusual B-cell lymphoma that presents as a malignant effusion, often in the
pleural cavity

Correct Answer. b

(100). Most important inheritance pattern of hereditary spherocytosis is

a. Autosomal dominant

b. Autosomal recessive

c. X linked dominant

d. X linked recessive

Solution. A
An autosomal dominant inheritance pattern is seen in about 75% of cases. The remaining patients have a more severe form of the disease
that is usually caused by the inheritance of two different defects (a state known as compound heterozygosity).

Correct Answer. a

Copyright © 2014 Delhi Academy of Medical Sciences, All Rights Reserved. 28/55
(101). The autoimmune haemolytic anemia seen in SLE is caused primarily by which kind of antibodies?

a. IgM

b. IgG

c. IgA

d. IgD

Solution. B

Correct Answer. b

(102). All of the following are included in diagnostic criteria for severe aplastic anemia except

a. Bone marrow cellularity <30%

b. Peripheral blood neutrophil count <0.5 x 109/L

c. Peripheral blood platelet count <20 x 109/L

d. Peripheral blood reticulocyte count <20 x 109/L

Solution. A

Correct Answer. a

(103). All of the following are intermediate filaments of cytoskeleton except

a. Laminin

b. Desmin

c. Cytokeratin

d. Vimentin

Solution. A
Intermediate filaments are 10-nm diameter fibrils that comprise a large and heterogeneous family. Individual types have characteristic
tissue-specific patterns of expression that can be useful for assigning a cell of origin for poorly differentiated tumors.
• Lamin A, B, and C: nuclear lamina of all cells
• Vimentin: mesenchymal cells (fibroblasts, endothelium)
• Desmin: muscle cells, forming the scaffold on which actin and myosin contract
• Neurofilaments: axons of neurons, imparting strength and rigidity
• Glial fibrillary acidic protein: glial cells around neurons
• Cytokeratins: at least 30 distinct varieties, subdivided into acidic (type I) and neutral/basic (type II); different types present in different
cells, hence can be used as cell markers

Correct Answer. a

(104). The most common mutations occurring in Marfan’s syndrome are

a. Deletions

b. Insertions

c. Missense

d. Nonsense

Solution. C
Mutational analysis has revealed more than 600 distinct mutations of the FBN1 gene in individuals with Marfan syndrome. Most of these
are missense mutations that give rise to abnormal fibrillin-1. These can inhibit polymerization of fibrillinfibers (dominant negative effect).

Correct Answer. c

(105). What percentage of genes on the inactivated X chromosome remain active?

Copyright © 2014 Delhi Academy of Medical Sciences, All Rights Reserved. 29/55
a. 21% on Xq, 3% on Xp

b. 21% on Xp, 3% on Xq

c. 12% on Xp, 3% on Xq

d. 12% on Xq, 3% on Xp

Solution. B
Molecular studies suggest that 21% of genes on Xp, and a smaller number (3%) on Xq escape X inactivation. At least some of the genes
that are expressed from both X chromosomes are important for normal growth and development

Correct Answer. b

(106). GAA triplet repeats are seen in

a. Fragile X syndrome

b. Friedrich ataxia

c. Huntington chorea

d. Fragile X tremor ataxia

Solution. B

Correct Answer. b

(107). Following is a flow cytometry graph of a patient complaining of fatigue and dark coloured urine. The normal red cell population is
coloured black while the abnormal red cells appear black on the graph. What is your diagnosis?

a. Pure erythroid leukemia

b. Paroxysmal cold hemoglobinuria

c. Aplastic anemia

d. Paroxysmal nocturnal hemoglobinuria

Solution. D
Reference: Page 642, Robbins and Cotran’s Pathological Basis of Disease, 9th edition
PNH blood cells are deficient in three GPI-linked proteins that regulate complement activity: (1) decayaccelerating factor, or CD55; (2)
membrane inhibitor of reactive lysis, or CD59; and (3) C8 binding protein. Of these factors, the most important is CD59, a potent inhibitor
of C3 convertase that prevents the spontaneous activation of the alternative complement pathway.

Correct Answer. d

(108). Fresh frozen plasma is separated from whole blood within how many hours after collection

a. 4 hours

b. 6 hours

c. 8 hours

d. 10 hours

Solution. C
The most familiar product for many blood centres is fresh frozen plasma (FFP), which is separated from whole blood and placed in a
freezer within eight hours of collection. Generally, ifplasma is frozen within 24 hours ofcollection, it can be labelled either as plasma
frozen within 24 hours (PF24) or plasma frozen within 24 hours after phlebotomy held at room temperature up to 24 hours after
phlebotomy (PF24RT24), although specific manufacturing requirements must be met in order to qualify for these labels. FFP, PF24 and
PF24RT24 can generally be maintained in a freezer at <-18 deg C for 12 months after collection.

Correct Answer. c

(109). All are vitamin K dependent coagulation factors except

a. Hageman factor

Copyright © 2014 Delhi Academy of Medical Sciences, All Rights Reserved. 30/55
b. Christmas factor

c. Factor X

d. Factor VII

Solution. A
Surface bound zymogens
Factor XII (Hageman factor)
Prekallikrein (Fletcher factor)
Factor XI
Vitamin K dependent zymogens (phospholipid bound)
Factor X
Factor IX (Christmas factor)
Factor VII
Factor II (Prothrombin)
Protein C (acts as an inhibitor when activated)
Cofactors or substrates
HMWK (Fitzgerald factor or Williams factor)
Factor VIII (antihemophilic factor)
Factor V
Fibrinogen
Protein S (Vitamin K dependent cofactor for activated Protein C)

Correct Answer. a

(110). The following will be positive in the given cell

a. Non specificesterase, CD 14, CD 64

b. Periodic acid schiff, CD 71, CD 235

c. Periodic acid schiff, CD 41, CD 61

d. Myeloperoxidase, CD 13, CD 33, CD 117

Solution. D
The photo is that of a myeloblast with Auer rods and hence option D is most suitable.

Correct Answer. d

(111). All of the following can show raised aPTT except

a. Disseminated intravascular coagulation

b. Von Willebrand disease

c. Bernard Soulier syndrome

d. Haemophilia A

Solution. C

Correct Answer. c

(112). Which of the following is the most important predictive factor in breast carcinoma?

a. Expression of hormone receptors

b. Ki 67 labelling index

c. Lymphatic node status

d. Tumor grade

Solution. A
Prognostic factors are used to assess the prognosis at the time of diagnosis. Predictive factors correlate with response to a specific
treatment. All the choices are prognostic of which lymph node status is most important. Expression of ER/PR is a predictive factor, useful
is predicting the response to treatment. Measurement of Her 2 neu gene has been recently accepted in clinical practice to select patients
for treatment with monoclonal antibodies.

Copyright © 2014 Delhi Academy of Medical Sciences, All Rights Reserved. 31/55
Correct Answer. a

(113). Which of the following is false about Robertsonian translocation?

a. Occurs between acrocentric chromosomes

b. Loss of the small product formed

c. Incompatible with life

d. Seen in 4% cases with Down’s syndrome

Solution. C
• A Robertsonian translocation (or centric fusion) is a translocation between two acrocentric chromosomes
• Typically the breaks occur close to the centromeres of each chromosome
• Transfer of the segments then leads to one very large chromosome and one extremely small one. Usually the small product is lost
• However, because it carries only highly redundant genes (e.g., ribosomal RNA genes), this loss is compatible with a normal phenotype.
• Robertsonian translocation between two chromosomes is encountered in 1 in 1000 apparently normal individuals.
• The significance of this form of translocation also lies in the production of abnormal progeny, as 4% cases with Down syndrome show
Robertsonian translocation.

Correct Answer. c

(114). Lymphoplasmacytic lymphoma is associated with

a. Hyperviscosity due to IgG antibodies

b. Bone destruction

c. Myd88 gene

d. CD 20 negative

Solution. C
• Lymphoplasmacytic lymphoma is a B-cell neoplasm of older adults that usually presents in the sixth or seventh decade of life.
• Although bearing a superficial resemblance to CLL/SLL, it differs in that a substantial fraction of the tumor cells undergo terminal
differentiation to plasma cells.
• Most commonly, the plasma cell component secretes monoclonal IgM, often in amounts sufficient to cause a hyperviscosity syndrome
known as Waldenströmmacroglobulinemia.
• Unlike multiple myeloma, complications stemming from the secretion of free light chains (e.g., renal failure and amyloidosis) are
relatively rare
• Bone destruction does not occur
• The lymphoid component expresses B-cell markers such as CD20 and surface Ig, whereas the plasma cell component secretes the same
Ig that is expressed on the surface of the lymphoid cells. This is usually IgM but can also be IgG or IgA.

Correct Answer. c

(115). Which of the following plays role in chemotaxis

a. LTB4

b. LTC4

c. LTD4

d. LTE4

Solution. (a) LTB4


Ref: Read the text below
Sol:
• LTb4 causes chemotaxis and other leukotrines are C4,D4 and E4 cause increased vascular permeability

Correct Answer. a

(116). A 60-year-old emaciated man is brought to the emergency with a distended abdomen, jaundice, ascites, and a slightly enlarged liver and
spleen. Laboratory studies show elevated levels of blood ammonia. A liver biopsy demonstrates cirrhosis. Histologic examination of brain
tissue from this patient would most likely show prominent changes in the nuclei of which of the following cell types?

a. Astrocytes

b. Ependymal lining cells

Copyright © 2014 Delhi Academy of Medical Sciences, All Rights Reserved. 32/55
c. Microglia

d. Neurons

Solution. (a) Astrocytes


Ref: Read the text below
Sol:
• Hepatic encephalopathy is a common clinical expression of liver failure, manifested as delirium, seizures, and coma.
• In general, the clinical symptoms greatly exceed their morphologic correlates, which are restricted to the appearance of altered
astroglia (termed Alzheimer type II astrocytes), which show enlarged nuclei and marginated chromatin, especially in the thalamus.
• The development of hepatic encephalopathy is caused by increased serum concentrations of neurotoxic substances, including ammonia.
• None of the other choices include cells that are altered by hepatic encephalopathy.

Correct Answer. a

(117). Deficiency of which of the following cytokine is associated with reduced thymic and lymphoid cellularity

a. IL-5

b. GM-CSF

c. IL-7

d. IL-3

Solution. C

Correct Answer. c

(118). Which of the following is not a component of alpha granules of platelets?

a. Fibrinogen

b. Von Willebrand factor

c. Factor V

d. ADP

Solution. D
α-Granules have the adhesion molecule Pselectin on their membranes (Chapter 3) and contain proteins involved in coagulation, such as
fibrinogen, coagulation factor V, and vWF, as well as protein factors that may be involved in wound healing, such as fibronectin, platelet
factor 4 (a heparinbinding chemokine), plateletderived growth factor (PDGF), and transforming growth factor-β. Dense (or δ) granules
contain adenosine diphosphate (ADP) and adenosine triphosphate, ionized calcium, serotonin, and epinephrine.

Correct Answer. d

(119). A 67year old man presents with increasing fatigue and is found to be anemic. He also complains of lower back ache. Physical examination
reveals a hard 1 cm nodule in the left lobe of the prostate and an increased Prostate Specific Antigen level. Examination of the peripheral
smear shows the following. Which of the following is the most likely diagnosis?

a. Megaloblastic anemia

b. Fanconi anemia

c. Myelophthisic anemia

d. Microangiopathichemolytic anemia

Solution. C
The picture shows tear drop cells and RBC precursors indicative of a fibrotic process in the marrow showing leucoerythroblastic reaction
in peripheral blood. The term myelophthisic anemia is best reserved for marrow replacement by nonhematologic tumors and
nonhematopoietic tissue. Minimal to moderate involvement usually does not cause symptoms or hematologic changes. Such infiltration is
clinically significant, however, because in patients with an established diagnosis of cancer, it indicates metastatic dissemination of the
tumor and usually an advanced stage. Although extensive infiltration may lead to anemia or even pancytopenia, anemia can be frequently
accompanied by an elevated leukocyte count, often with immature myeloid cells in the blood. Platelets can be increased, decreased, or
normal (megakaryocytic fragments are seen occasionally in the blood film). The findings accompanied by teardrop-shaped red cells
(dacrocytes), prematurely released nucleated red cells, and immature myeloid cells is referred to as leukoerythroblastic reaction, which
generally reflects marrow replacement by tumor or extramedullary hematopoiesis.

Correct Answer. c

Copyright © 2014 Delhi Academy of Medical Sciences, All Rights Reserved. 33/55
(120). Cancer cachexia is believed to be mediated by

a. TNF

b. IL-1

c. TGF beta

d. IL-12

Solution. A
The precise causes of cachexia are not known, but it is clear that mediators secreted by tumors and during chronic inflammatory
reactions contribute to its development:
• Proteolysis-inducing factor, which is a glycosylated polypeptide excreted in the urine of weight-losing patients with pancreatic, breast,
colon, and other cancers
• Lipid-mobilizing factor, which increases fatty acid oxidation, and proinflammatory cytokines, such as TNF (originally known as
cachectin), and IL-6.

Correct Answer. a

(121). Which phenomenon/phenomena characterise irreversibility of cell injury?

a. Mitochondrial dysfunction

b. Profound membrane damage

c. Nuclear disaggregation

d. Both A and B

Solution. D
Two phenomena consistently characterize irreversibility—the inability to reverse mitochondrial dysfunction (lack of oxidative
phosphorylation and ATP generation) even after resolution of the original injury, and profound disturbances in membrane function. Injury
to lysosomal membranes results in the enzymatic dissolution of the injured cell that is characteristic of necrosis.

Correct Answer. d

(122). All of the following are pro apoptotic except

a. Smac

b. Diablo

c. BAX

d. BCL- XL

Solution. D
• Smac/Diablo, enter the cytoplasm, where they bind to and neutralize cytoplasmic proteins that function as physiologic inhibitors of
apoptosis (called IAPs).
• The normal function of the IAPs is to block the activation of caspases, including executioners like caspase-3, and keep cells alive. Thus,
the neutralization of these IAPs permits the initiation of a caspase cascade.
• BAX and BAK are the two prototypic members of the BCL2 family.
• BCL XL, MCL1 and BCL2 are the anti apoptotic members

Correct Answer. d

(123). Dilated sinusoids and irregular cystic spaces filled with blood within the liver, which may rupture, leading to massive
intraabdominalhemorrhage,are most commonly associated with

a. Salicylates

b. Estrogens

c. Anabolic steroids

d. Acetaminophen

Copyright © 2014 Delhi Academy of Medical Sciences, All Rights Reserved. 34/55
Solution. (c) Anabolic steroids.
Ref– Read the text below
Sol:
• Hepatic injury can result from a wide range of drugs, chemicals, and toxins. Peliosishepatis is an abnormality of the hepatic blood flow
that results in sinusoidal dilation and the formation of irregular blood-filled lakes, which may rupture and produce massive
intraabdominalhemorrhage or hepatic failure.
• Peliosis hepatitis is most often associated with the use of anabolic steroids, but more rarely it may be associated with oral
contraceptives.
• Reye’s syndrome,characterized by microvesicular fatty change in the liver and encephalopathy,has been related to the use of salicylates
in children with viral illnesses.
• Acetaminophen toxicity results in centrilobular liver necrosis, while estrogens may be related to thrombosis of the hepatic or portal
veins.
• Several hepatic tumors are related to exposure to vinyl chloride, including angiosarcoma and hepatocellular carcinoma.

Correct Answer. c

(124). Richter transformation is usually seen in

a. Burkitt lymphoma

b. Follicular lymphoma

c. Small lymphocytic lymphoma

d. Diffuse large B cell lymphoma

Solution. C
SLL/CLL is a small cell low grade lymphoma which can undergo Richter transformation to large cell high grade lymphoma like DLBCL

Correct Answer. c

(125). Pautrier’smicroabscesses are seen in

a. Lichen planus

b. Psoriasis

c. Pemphigus vulgaris

d. Mycoses fungoides

Solution. D
Collections of tumor lymphocytes within the epidermis are called Pautrier’smicroabscesses.

Correct Answer. d

(126). The following immunofluorescence picture is most likely seen in which of the following blistering disorders?

a. Pemphigus vulgaris

b. Pemphigus foliaceous

c. Bullous pemphigoid

d. Dermatitis herpetiformis

Solution. C
• Bullous pemphigoid is caused by autoantibodies that bind to proteins that are required for adherence of basal keratinocytes to the
basement membrane
• Most antibody deposition occurs in a continuous linear pattern at the dermoepidermal junction, which contains specialized structures
called hemidesmosomes that link basal keratinocytes to the underlying basement membrane
• The so-called bullous pemphigoid antigens (BPAGs) are components of hemidesmosomes.
• Antibodies against one such component called BPAG2 are proven to cause blistering.

Correct Answer. c

(127). Which of the following is false about paroxysmal cold hemoglobinuria?

a. Erythrophagocytosis is a common feature

Copyright © 2014 Delhi Academy of Medical Sciences, All Rights Reserved. 35/55
b. Donath Landsteiner antibody associated

c. IgM antibody mediated

d. Seen following viral infections

Solution. C
• PCH is an autoimmune hemolytic syndrome most often seen in children following infection by mumps, chickenpox, measles, and other
viruses
• Historically, PCH was also seen in syphilitic patients
• It can present suddenly with severe hemolysis with an initial Hg of 7 g/dL or lower
• In addition to laboratory findings of intravascular hemolysis, PCH patients may have erythrophagocytosis on peripheral blood smear
• The latter is highly unusual and should trigger an investigation for possible PCH
• AutoantiP, also known as the DonathLandsteiner (DL) antibody, is the most common causative antibody in PCH
• It is an IgG, biphasic autohemolysin capable of binding to RBCs at cold temperatures and causing intravascular hemolysis of those cells
at body temperature.

Correct Answer. c

(128). A patient with complaints of hematuria, proteinuria, edema and hypertension is being evaluated. Serum ANCA positivity is found. During
the course of evaluation, patient worsens with oliguria. Electron microscopy performed on the renal biopsy shows the following. What will
be the pathology seen in glomerulus on histopathology?

a. Membanous nephropathy

b. Membranoproliferative glomerulonephritis

c. Focal segmental glomerulosclerosis

d. Crescentric glomerulonephritis

Solution. D
th th
Reference: Robbins and Cotran’s Pathological Basis of Disease, 9 edition and Basic Pathology, 10 edition
Electron micrograph shows characteristic wrinkling of glomerular basement membrane with focal disruptions, seen in RPGN.

Correct Answer. d

(129). All are true about autosomal recessive inheritance except

a. Incomplete penetrance

b. Recurrence risk is 25% for each birth

c. Usually parents are unaffected

d. Usually include inborn errors of metabolism

Solution. A
Autosomal recessive traits make up the largest category of Mendelian disorders. They occur when both alleles at a given gene locus are
mutated. These disorders are characterized by the following features: (1) The trait does not usually affect the parents of the affected
individual, but siblings may show the disease; (2) siblings have one chance in four of having the trait (i.e., the recurrence risk is 25% for
each birth); and (3) if the mutant gene occurs with a low frequency in the population, there is a strong likelihood that the affected
individual (proband) is the product of a consanguineous marriage. The expression of the defect tends to be more uniform than in
autosomal dominant disorders.
• Complete penetrance is common.
• Onset is frequently early in life.
• Although new mutations associated with recessive disorders do occur, they are rarely detected clinically. Since the individual with a
new mutation is an asymptomatic heterozygote, several generations may pass before the descendants of such a person mate with other
heterozygotes and produce affected offspring.
• Many of the mutated genes encode enzymes. In heterozygotes, equal amounts of normal and defective enzyme are synthesized. Usually
the natural “margin of safety” ensures that cells with half the usual complement of the enzyme function normally

Correct Answer. a

(130). The antibody seen with neonatal lupus is

a. Anti DNA

b. AntiSmith antigen

c. Anti Ro

Copyright © 2014 Delhi Academy of Medical Sciences, All Rights Reserved. 36/55
d. Anti histone

Solution. C

Correct Answer. c

(131). Gold standard investigation for Paroxysmal nocturnal hemoglobinuria is

a. Ham test

b. Sucrose lysis test

c. Flow cytometry

d. Immunohistochemistry

Solution. C

Correct Answer. c

(132). Mode of inheritance of paroxysmal nocturnal hemoglobinuria is

a. X linked recessive

b. Autosomal recessive

c. Autosomal dominant

d. None of the above

Solution. D
PNH is an acquired hematopoetic stem cell defect, it has no mode of inheritance

Correct Answer. d

(133). A 33-year-old woman presents with a diffuse scaly skin rash of 4 weeks duration. Biopsy of lesional skin reveals a cutaneous T-cell
lymphoma (mycosis fungoides). Which of the following immunohistochemical markers would be most useful for identifying malignant cells
in the skin of this patient?

a. Calcitonin

b. CD4

c. Desmin

d. S-100

Solution. (b) CD4


Ref: Read the text below
Sol:
• CD4 is a cluster-differentiation antigen of helper T lymphocytes.
• HMB-45 and S-100 are markers for malignant melanoma, among other tumors.
• Calcitonin is a peptide hormone.
• Desmin is an intermediate filament protein found in cells of mesenchymal origin.

Correct Answer. b

(134). Councilman bodies found in acute viral hepatitis are example of

a. Atrophy

b. Necrosis

c. Apoptosis

d. Reversible cell injury

Copyright © 2014 Delhi Academy of Medical Sciences, All Rights Reserved. 37/55
Solution. (c) Apoptosis
Ref: Read the text below
Sol:
• Councilman bodies and pathological atrophy secondary to obstruction are few examples of pathological apoptosis

Correct Answer. c

(135). Most common non Hodgkin Lymphoma worldwide is

a. CLL/SLL

b. DLBCL

c. Follicular lymphoma

d. Mantle cell lymphoma

Solution. B
Refrerence: WHO Classification of Tumors of Hematopoetic and Lymphoid tissues, 4th edition (2017)

Correct Answer. b

(136). A 23-year-old female presents with pelvic pain and is found to have an ovarian mass of the left ovary that measures 3 cm in diameter.
Grossly, the mass consists of multiple cystic spaces. Histologically, these cysts are lined by tall columnar epithelium, with some of the
cells being ciliated. What is your diagnosis of this ovarian tumor, which histologically recapitulates the histology of the fallopian tubes?

a. Serous tumor

b. Mucinous tumor

c. Endometrioid tumor

d. Clear cell tumor

Solution. (a) Serous tumor


Reference: Read the text below
Sol:
• The surface epithelial tumors of the ovary are derived from the surface celomic epithelium, which embryonically gives rise to the
Müllerian epithelium. Therefore these ovarian epithelial tumors may recapitulate the histology of organs derived from the Müllerian
epithelium. For example, serous ovarian tumors are composed of ciliated columnar serous epithelial cells, which are similar to the lining
cells of the fallopian tubes.
• Endometrioid ovarian tumors are composed of nonciliated columnar cells, which are similar to the lining cells of the endometrium.
• Mucinous ovarian tumors are composed of mucinous nonciliated columnar cells, which are similar to the epithelial cells of the
endocervical glands. Other epithelial ovarian tumors are similar histologically to other organs of the urogenital tract, such as the clear
cell ovarian carcinoma and the Brenner tumor.
• Clear cell carcinoma of the ovary is similar histologically to clear cell carcinoma of the kidney, or more accurately,the clear cell variant
of endometrial adenocarcinoma or the glycogen-rich cells associated with pregnancy.
• The Brenner tumor is similar to the transitional lining of the renal pelvis or bladder. This ovarian tumor is associated with benign
mucinous cystadenomas of the ovary.

Correct Answer. a

(137). CD 10 positive lymphomas may include all except

a. Diffuse large B cell lymphoma

b. Burkitt lymphoma

c. Mantle cell lymphoma

d. Follicular lymphoma

Solution. C
CD 10 is usually seen in lymphomas arising from germinal centre B cells. Mantle cell lymphoma does not arise from a germinal centre B
cell and hence is CD 10 negative.

Correct Answer. c

(138). Which of the following is/are an example of non megaloblastic macrocytic anemia?

a. Alcoholism

Copyright © 2014 Delhi Academy of Medical Sciences, All Rights Reserved. 38/55
b. Liver disease

c. Hypothyroidism

d. All of the above

Solution. D

Correct Answer. d

(139). A 2-year-old boy is found to have bilateral retinal tumors. Molecular studies demonstrate a germline mutation in one allele of the Rbgene.
Which of the following genetic events best explains the mechanism of carcinogenesis in this patient?

a. Balanced translocation

b. Expansion of trinucleotide repeat

c. Gene amplification

d. Loss of heterozygosity

Solution. (d) Loss of heterozygosity


Ref: Read the text below
Sol:
• Retinoblastomas are malignant ocular tumors of young children.
• In cases of hereditary retinoblastoma, an affected child inherits one defective Rb allele together with one normal gene.
• This heterozygous state is not associated with any observable changes in the retina because 50% of the Rbgene product is sufficient to
prevent the development of retinoblastoma.
• However, if the remaining normal Rballele is inactivated by deletion or mutation, the loss of its suppressor function leads to the
appearance of a neoplasm.
• This genetic process is referred to as loss of heterozygosity.
• The other choices have not been associated with the loss of tumor suppressor genes in somatic cells.

Correct Answer. d

(140). A 28-year-old man presents with 6 days of fever and shortness of breath. His temperature is 38.7°C (103°F), respirations are 30 per
minute, and blood pressure is 120/80 mm Hg. A chest X-ray reveals diffuse interstitial and alveolar infiltrates. Sputum cultures are
negative, and the patient does not respond to standard antibiotic therapy. A transbronchial lung biopsy is shown in the image. Which of
the following is the appropriate diagnosis?

a. Eosinophilic pneumonia

b. Lipoid pneumonia

c. Pneumococcal pneumonia

d. Pneumocystis pneumonia

Solution. (a) Eosinophilic pneumonia


Ref: Read the text below
Sol:
• Eosinophilic pneumonia is principally an allergic disorder.
• It refers to the accumulation of eosinophils in alveolar spaces and is classified as either idiopathic or secondary to an underlying illness.
• In acute eosinophilic pneumonia, the alveolar spaces are filled with an inflammatory exudate composed of eosinophils and macrophages.
• The alveolar septa are thickened by the presence of numerous eosinophils and hyaline membranes are present.
• Patients respond dramatically to corticosteroids, and, in contrast to chronic eosinophilic pneumonia, acute eosinophilic pneumonia does
not recur.
• Excess eosinophils are not encountered in the other choices.

Correct Answer. a

(141). All of the following deficiencies lead to asymptomatic presentations except

a. Factor XII

b. Prekallikrein

c. High molecular weight kininogen

d. Factor II

Copyright © 2014 Delhi Academy of Medical Sciences, All Rights Reserved. 39/55
Solution. D
Reference: Wintrobe’s Clinical Hematology, 14th edition (2018)
Deficiencies of factors XII, PK, and HK are not associated with bleeding, even after trauma or surgery.

Correct Answer. d

(142). Prolonged PT and aPTT can be seen in all of the following conditions except

a. Factor V deficiency

b. Disseminated intravascular coagulation

c. Factor XI deficiency

d. Factor X deficiency

Solution. C
th
Reference: Wintrobe’s Clinical Hematology, 14 edition (2018)
Prolongation of both PT and aPTT indicates involvement of both pathways/common pathway. Since factor XI is a part of only intrinsic
pathway, its deficiency leads to increase in aPTT but not PT.

Correct Answer. c

(143). The best anti-coagulant for performing osmotic fragility test is

a. Heparin

b. EDTA

c. Double oxalate

d. Trisodium citrate

Solution. A
Heparin
- lithium or sodium salt
- commonly used for chemistry, gas analysis and emergency tests
- does not alter size of red cells
- best used for OFT
- not suitable for blood counts as it induces platelet and leucocyte clumping
- not used to make smears as it gives a blue discoloration with Romanowsky dyes
- not used in PCR as it inhibit enzyme activity

Correct Answer. a

(144). A 50-year-old woman presents with a slowly enlarging, painful mass at the angle of the right jaw. A biopsy of the mass is shown in the
image. Which of the following is the most likely diagnosis?

a. Acinic cell carcinoma

b. Adenoid cystic carcinoma

c. Malignant mixed tumor

d. Malignant oncocytoma

Solution. (b) Adenoid cystic carcinoma


Ref: Read the text below
Sol:
• Adenoid cystic carcinoma is a slowly growing malignant neoplasm of the salivary gland that is notorious for its tendency to invade
locally and to recur after surgical resection.
• The tumor cells are small, have scant cytoplasm, and grow in solid sheets or as small groups, strands, or columns.
• Within these structures,the tumor cells interconnect to enclose cystic spaces, resulting in a solid, tubular or cribriform (sieve-like)
arrangement.
• The other choices do not exhibit the typical cribriform pattern

Correct Answer. b

(145). Prothrombin time assesses activity of all of the following coagulation factors except

Copyright © 2014 Delhi Academy of Medical Sciences, All Rights Reserved. 40/55
a. V

b. X

c. XII

d. VII

Solution. (C) XII


The prothrombin time (PT) assay assesses the function of the proteins in the extrinsic pathway (factors VII, X, V, II, and fibrinogen). In
brief, tissue factor, phospholipids, and calcium are added to plasma and the time for a fibrin clot to form is recorded.
The partial thromboplastin time (PTT) assay screens the function of the proteins in the intrinsic pathway (factors XII, XI, IX, VIII, X, V, II,
and fibrinogen). In this assay, clotting of plasma is initiated by addition of negative charged particles (e.g., ground glass) that activate
factor XII (Hageman factor) together with phospholipids and calcium, and the time to fibrin clot formation is recorded

Correct Answer. c

(146). Which of the following is not a free radical scavanger

a. Cerruloplasmin

b. Transferrin

c. Copper

d. Glutathione peroxidase

Solution. C
th th
Reference: Robbins and Cotran Pathological Basis of Disease 9 edition and Basic Pathology 10 edition.
Free radical scavenging in the body is done in the following ways
- Antioxidants (vitamin E, A, ascorbic acid, glutathione)
- Ceruloplasmin, transferrin, ferritin, lactoferrin- storage and transport proteins which bind reactive metals like copper and iron and
prevent ROS formation
- Enzymes like SOD, glutathione peroxidase and catalase

Correct Answer. c

(147). All of the following are pro inflammatory except

a. PGE2

b. PGI2

c. LTB4

d. Lipoxins

Solution. D

Correct Answer. d

(148). Which of the following is not expressed by tumor cells of Langerhans cell histiocytosis?

a. CD 1a

b. HLA DR

c. CD 68

d. CD 20

Solution. D
Reference- WHO 2017
LCH consistently expresses CD1a, langerin (also called CD207), and S100 protein. Langerin and CD1a staining may be particularly useful
in detecting bone marrow involvement. In addition, the cells are positive for vimentin, CD68, and HLA-DR. CD45 expression and lysozyme
content is low. B-cell and T-cell lineage markers (except for CD4), CD30, and follicular dendritic cell markers are absent.

Correct Answer. d

(149). Definitive diagnosis of malignancy in pheochromocytoma is based on

Copyright © 2014 Delhi Academy of Medical Sciences, All Rights Reserved. 41/55
a. Capsular invasion

b. Vascular invasion

c. Cellular pleomorphism

d. Metastasis

Solution. D
th th
Reference: Robbins and Cotran’s Pathological Basis of Disease, 9 edition and Basic Pathology, 10 edition
Both capsular and vascular invasion, as well as cellular pleomorphism, may be encountered in some benign lesions. Therefore, the
definitive diagnosis of malignancy in pheochromocytomas is based on the presence of metastases. These may involve regional lymph
nodes as well as more distant sites, including liver, lung, and bone.

Correct Answer. d

(150). T cell rich area in lymph node is

a. Cortex

b. Medulla

c. Subcapsular sinus

d. Paracortex

Solution. D
Reference: Ioachim’s Lymph Node Pathology, 4th edition
1. Cortical area/superficial cortex-
- lymphoid follicles with their germinal centers
- bursa dependent B cells
- associated with mechanisms of humoral immunity
2. Lymphocytes in mantle zone are all of B type
3. Paracortical area/deep cortex-
- densely cellular area beneath the cortex
- forms interdigitations from the capsule to the corticomedullary junction
- thymus dependent area: Predominantly T cells
- post capillary venules
- also has interdigitating cells
4. Medulla-
- main site of plasma cell proliferation, differentiation and production of antibodies

Correct Answer. d

(151). Which of the following statements is true?

a. Actin is the most abundant cytosolic protein in the cell

b. Lamin is the most abundant glycoprotein in the basement membrane

c. Fibronectin imparts scaffolding to the cytoplasm

d. FACITs are seen in association with type I collagen

Solution. a
Laminin is the most abundant glycoprotein. Lamin is an intermediate filament of the cytosol.
Fibronectin is a component of the extra cellular matrix, not cytoplasm.
FACITs (fibril-associated collagen with interrupted triple helices), such as type IX collagen are in cartilage.

Correct Answer. a

(152). Squamous metaplasia is seen in all of the following except

a. Vitamin A deficiency

b. Smoking

c. Gastroesophageal reflux disease

d. Stone obstructing a duct

Copyright © 2014 Delhi Academy of Medical Sciences, All Rights Reserved. 42/55
Solution. C
Options A, B, D are examples of change of columnar epithelium to squamous. However GERD in esophagus causes change of squamous to
columnar (called Intestinal Metaplasia or Barret’s Esophagus)

Correct Answer. c

(153). Parahemophilia is another term used for

a. Prothrombin deficiency

b. Afibrinogenemia

c. Von Willebrand disease

d. Factor V deficiency

Solution. D
th
Reference: Wintrobe’s Clinical Hematology, 14 edition (2018)
Inherited factor V deficiency, also known as labile factor deficiency, proaccelerin deficiency, and parahemophilia, is a rare autosomal
recessive bleeding disorder first described by Owren. To date, there are over 200 reported cases, with an estimated frequency of 1 and a
database at isth.org lists >120 factor V mutations. As with some of the other factor deficiencies, the frequency of factor V deficiency is
greater in countries where consanguineous marriages occur

Correct Answer. d

(154). Which of the following is associated with action on or in the apical or luminal surface of the intestinal mucosal cell?

a. Hepcidin

b. Ferroportin

c. Haphestin

d. DMT1

Solution. D
Reference: Wintrobe’s Clinical Hematology, 14th edition (2018)

Correct Answer. d

(155). All are true about DMT1 deficiency associated Iron deficiency anemia except

a. Liver iron increased

b. Serum iron increased

c. High transferrin saturation

d. High hepcidin levels

Solution. D

Correct Answer. d

(156). Graft vs host disease is mediated by

a. Host lymphocytes against donor antigens

b. Donor lymphocytes against host antigens

c. Both

d. Donor lymphocytes against both host and graft antigens

Copyright © 2014 Delhi Academy of Medical Sciences, All Rights Reserved. 43/55
Solution. B
GVHD occurs when immunologically competent cells or their precursors are transplanted into immunologically crippled recipients, and
the transferred cells recognize alloantigens in the host and attack host tissues. When immune-compromised recipients receive HSC
preparations from allogeneic donors, the immunocompetent T cells present in the donor inoculum recognize the recipient’s HLA antigens
as foreign and reactagainst them. To try to minimize GVHD, HSC transplants are done between donor and recipient that are HLAmatched
using precise DNA sequencing-based methods for molecular typing of HLA alleles.

Correct Answer. b

(157). Which are immune changes seen in AIDS

a. Loss of CD 4 + T helper cell

b. Decreased delayed type hypersensitivity

c. Decreased IL-2 and IFN gamma

d. All of the above

Solution. D

Correct Answer. d

(158). Which of the listed coagulation disorders is this pedigree chart most likely to correspond to

a. Bernard Soulier syndrome

b. Von Willebrand disease

c. Hemophilia A

d. Glanzmann thrombasthenia

Solution. C
The pedigree chart is of X linked recessive trait and the most suitable answer for the same is C

Correct Answer. c

(159). A 25year old woman with complaints of malar rash, arthritis and anemia since three months is being investigated. Peripheral smear
shows the following. The next investigation should be

a. Osmotic fragility test

b. Sickling test

c. HPLC

d. Coomb’s test

Solution. D
Short history of anemia, malar rash, arthritis in a young female should raise the suspicion of SLE. Spherocytes in the peripheral smear
point to the possibility of AIHA, hence Coomb’s test should be the next step of investigation.

Correct Answer. d

(160). Most common symptom of plasma cell myeloma is

a. Bone pain

b. Anemia

c. Infections

d. Hyperviscosity

Solution. A
Reference: Harrison’s Principles of Internal Medicine, 20th edition (2018)

Copyright © 2014 Delhi Academy of Medical Sciences, All Rights Reserved. 44/55
Correct Answer. a

(161). The following peripheral smear picture can be seen in all of the following except

a. Disseminated intravascular coagulation

b. Hemolytic uremic syndrome

c. Hereditary spherocytosis

d. Thrombotic thrombocytopenic purpura

Solution. C
Picture shows fragmented RBCs or schistocytes.

Correct Answer. c

(162). Which is true about apoptosis?

a. Not physiological

b. Inflammatory reaction is present

c. Councilman bodies are sign of apoptosis

d. Cell membrane is damaged

Solution. C

Correct Answer. c

(163). Which of the following contributes to cancer progression?

a. Autophagy

b. Apoptosis

c. Senescence

d. Angiogenesis

Solution. D
Angiogenesis encompasses a cascade of events that allow a tumor’s vascular supply to be enhanced thereby increasing cellular
proliferation.

Correct Answer. d

(164). Which of the following Acute myeloid leukemia presents with abnormally granular eosinophils?

a. AML with t(8;21)

b. AML with t(9;22)

c. AML with inv(16)

d. AML with t(15;17)

Solution. C
Reference: WHO Classification of Tumors of Hematopoeitic and Lymphoid tissues (2017) In AML with inv(16)(p13.1q22) or t(16;16)
(p13.1;q22), in addition to the usual morphological features of acute myelomonocytic leukaemia, the bone marrow shows a variable
number of eosinophils (usually increased, but sometimes <5%) at all stages of maturation, without significant maturation arrest. The most
striking abnormalities involve the immature eosinophilic granules, mainly evident at the late promyelocyte and myelocyte stages. The
abnormalities are usually not present at later stages of eosinophil maturation. The eosinophilic granules are often larger than those
normally present in immature eosinophils, are purple-violet in colour, and in some cells are so dense that they obscure the cell
morphology. The mature eosinophils occasionally show nuclear hyposegmentation.

Correct Answer. c

Copyright © 2014 Delhi Academy of Medical Sciences, All Rights Reserved. 45/55
(165). Red cell distribution width is a measure of

a. Poikilocytosis

b. Microcytosis

c. Anisocytosis

d. Macrocytosis

Solution. C
th
Reference: Wintrobe’s Clinical Hematology. 14 edition (2018)
The RDW is a quantitative measure of the variation in red cell size also called anisocytosis; and the higher the values, the more
heterogeneous the RBC population size.

Correct Answer. c

(166). Myd88 mutation is characteristically seen in

a. Lymphoplasmacytic lymphoma

b. Plasma cell lymphoma

c. IgG/IgA monoclonal gammopathy of uncertain significance

d. Smoldering myeloma

Solution. A

Correct Answer. a

(167). Starry sky appearance can be seen in all except

a. Burkitt lymphoma

b. Diffuse large B cell lymphoma

c. Acute lymphoblastic lymphoma

d. Chronic lymphocytic lymphoma

Solution. D
Starry sky appearance is a feature of high grade mitotically active lymphomas. CLL is a low grade lymphoma

Correct Answer. d

(168). A patient with known sickle cell disease registers with a new physician. CBC ordered shows macrocytosis of RBCs. The most likely
explanation is

a. Iron deficiency

b. B12 deficiency

c. Folate deficiency

d. Patient is on hydroxyurea

Solution. D

Correct Answer. d

(169). Which of the following malignancies does not have a significant increase in incidence among people with Lynch syndrome?

a. Colon cancer

b. Cervical cancer

c. Ovarian cancer

d. Endometrial cancer

Copyright © 2014 Delhi Academy of Medical Sciences, All Rights Reserved. 46/55
Solution. B
The Lynch syndrome is primarily associated with a high risk for colon and endometrial cancer and an increased risk for other tumors like
ovary, stomach, small intestine, hepatobiliary tract, urinary tract, brain and skin. Cervical cancer is one of the known associated cancer
types.

Correct Answer. b

(170). Which of the following Acute lymphoblastic leukemia has good prognosis?

a. B-ALL, BCR-ABL1 like

b. B-ALL with KMT2A rearrangement

c. B-ALL with t(12;21)

d. B-ALL with hypoploidy

Solution. C

Correct Answer. c

(171). Which of the following clinical manifestations is seen in thrombocytopenia and qualitative platelet disorders

a. Petechiae

b. Hemarthrosis

c. Deep hematomas

d. Delayed bleeding

Solution. A
Platelet type disorders will generally manifest as mucosal bleeding (petechiae, purpurae), moderate bleeding from small wounds and
heavy menses. The remaining answer choices represent physical findings caused by coagulation disorders

Correct Answer. a

(172). Which of the following variants of DLBCL are not associated with EBV infection

a. DLBCL associated with chronic inflammation

b. Intravascular large B cell lymphoma

c. Primary effusion lymphoma

d. Lymphomatoid granulomatosis

Solution. B

Correct Answer. b

(173). Neoplastic mast cells are positive for

a. CD 25

b. CD 117

c. CD 3

d. CD 20

Solution. A
A> B

Correct Answer. a

(174). Most common glycolytic enzyme deficiency in RBCs is

Copyright © 2014 Delhi Academy of Medical Sciences, All Rights Reserved. 47/55
a. Pyruvate kinase

b. Glucose 6 phosphate dehydrogenase

c. Phosphofructokinase

d. Aldolase

Solution. A
th
Reference: Wintrobe’s Clinical Hematology. 14 edition (2018)
Of the enzymatic deficiencies involving glycolysis, PK deficiency is the most common and the most extensively studied.

Correct Answer. a

(175). All of the following is true about cold autoantibodies against RBC antigens in autoimmune hemolyticanemia except

a. Little activity at body temperature

b. Usually IgG

c. Intravascular hemolysis

d. Greater affinity for RBCs as temperature decreases

Solution. B
Reference: Wintrobe’s Clinical Hematology. 14th edition (2018)
Cold-active antibodies typically have little, if any, activity at body temperature, but have greater affinity for RBCs as the temperature
decreases toward 0°C. Conversely, warm-active antibodies have their greatest affinity at 37°C. Generally speaking, cold-active antibodies
are typically immunoglobulin (Ig)-M, fix complement, and lead to immediate intravascular RBC destruction or hepatic-mediated clearance.
In contrast, warmactive antibodies are typically IgG, may or may not fix complement, and primarily lead to RBC loss by splenic-mediated
clearance of sensitized cells.

Correct Answer. b

(176). The Wilson's disease gene (ATP7B) has been mapped to:

a. Chromosome 11

b. Chromosome 13

c. Chromosome 17

d. Chromosome 21

Solution. (b)chromosome 13
Ref:Read the text below
Sol:
• The Wilson's disease gene (ATP7B) has been mapped to chromosome 13 (13q14.3) and is expressed primarily in the liver, kidney, and
placenta. The gene codes for a P-type (cation transport enzyme) ATPase that transports copper into bile and incorporates it into
ceruloplasmin.
• Mutations can be detected in 90% of patients. Most (60%) are homozygous for ATP7B mutations (two abnormal copies), and 30% have
only one abnormal copy. 10% have no detectable mutation.
• Although 300 mutations of ATP7B have been described, in most populations the cases of Wilson's disease are due to a small number of
mutations specific for that population. For instance, in Western populations the H1069Q mutation (replacement of a histidine by a
glutamine at position 1069 in the protein) is present in 37-63% of cases, while in China this mutation is very uncommon and R778L
(arginine to leucine at 778) is found more often.
• Relatively little is known about the relative impact of various mutations, although the H1069Q mutation seems to predict later onset and
predominantly neurological problems, according to some studies.

Correct Answer. b

(177). The earliest finding on peripheral smear in iron deficiency anemia is

a. Microcytosis

b. Pencil shaped cells

c. Hypochromia

d. Anisocytosis

Copyright © 2014 Delhi Academy of Medical Sciences, All Rights Reserved. 48/55
Solution. D
Reference: Wintrobe’s Clinical Hematology, 14th edition (2018)
Anisocytosis is an important early sign in iron deficiency and one that has differential diagnostic value when quantified. The red cell
distribution width (RDW) is increased in iron deficiency, and this often is useful in distinguishing iron deficiency from thalassemia trait
conditions in which the RDW is normal.

Correct Answer. d

(178). The first investigation to rise in a patient with hemochromatosis is

a. Transferrin saturation

b. Total iron binding capacity

c. Stainable liver iron

d. Ferritin

Solution. A
Reference: Wintrobe’s Clinical Hematology, 14th edition (2018)
Transferrin saturation, the first blood test to become elevated inhemochromatosis homozygotes, is typically >60% in symptomatic men
and >50% in symptomatic women. Total serum iron-binding capacity is often mildly reduced and unbound iron-binding capacity is usually
decreased in untreated homozygotes. In iron-loaded patients, serum ferritin concentration is elevated. Liver stainable iron and the hepatic
iron index are also elevated in ironloaded homozygotes. The sensitivity of elevated transferrin saturation to identify a patient with
hemochromatosis is 94% to 98%, and its specificity is 70% to 98%.

Correct Answer. a

(179). All of the following are integral membrane proteins of RBC except

a. Ankyrin

b. Band 3

c. Rh protein

d. Glycophorin

Solution. A
Reference: Wintrobe’s Clinical Hematology, 14th edition (2018)
Typically, membrane proteins are classified as integral, penetrating or crossing the lipid bilayer and interacting with the hydrophobic
lipid core, or peripheral, interacting with integral proteins or lipids at the membrane surface but not penetrating into the bilayer core.
The integral membrane proteins include the glycophorins, the Rh proteins, Kell and Duffy antigens, and transport proteins such as band 3
(AE1, anion exchanger 1), Na+ K+-ATPase, Ca2+-ATPase, and Mg2+ATPase. An assortment of membrane receptors and antigens are
present on various integral membrane proteins. Peripheral membrane proteins include the structural proteins of the spectrin–actin-based
membrane skeleton.

Correct Answer. a

(180). The following peripheral smear is from a patient with lymphocytosis, lymphadenopathy, anemia and splenomegaly. The CD markers
expected to be positive on flow cytometry on these atypical cells are

a. CD 5 + CD 23 –

b. CD 5 + CD 23 +

c. CD 5 – CD 10 + BCL 2 +

d. CD 5 – CD 10 + BCL 2 –

Solution. B
The picture shows monotonous small atypical lymphocytes with coarse chromatin, smudge cells, spherocytes and nucleated RBC
(suggestive of AIHA). Probable diagnosis is CLL and hence B is the most suitable profile.

Correct Answer. b

(181). Earliest change in reversible cell injury is

a. Depletion of ATP

Copyright © 2014 Delhi Academy of Medical Sciences, All Rights Reserved. 49/55
b. Influx of calcium

c. Cellular swelling

d. Myelin figures

Solution. A
Biochemical alterations are the earliest findings in cell injury hence A is the best answer.

Correct Answer. a

(182). Red neurons can be seen after ___ hours after neuronal injury

a. >4 hours

b. 12-24 hours

c. 48-72 hours

d. >72 hours

Solution. B
Acute neuronal injury (“red neurons”) refers to a spectrum of changes that accompany acute CNS hypoxia/ischemia or other acute insults
and reflect the earliest morphologic markers of neuronal cell death. “Red neurons” are evident by about 12 to 24 hours after an
irreversible hypoxic/ ischemic insult. The morphologic features consist of shrinkage of the cell body, pyknosis of the nucleus,
disappearance of the nucleolus, and loss of Nissl substance, with intense eosinophilia of the cytoplasm.

Correct Answer. b

(183). Limbic encephalitis mediated by ANNA1 antibody as a paraneoplastic syndrome is seen with

a. Adenocarcinoma of colon

b. Small cell lung carcinoma

c. Breast carcinoma

d. Prostate carcinoma

Solution. B
Limbic encephalitis is characterized by subacute dementia and marked by perivascular inflammatory cuffs, microglial nodules, some
neuronal loss, and gliosis, most evident in the anterior and medial portions of the temporal lobe; the microscopic picture resembles that of
an infectious process. A comparable process involving the brainstem can be seen in isolation or together with limbic system involvement.
Some of these patients have a circulating ANNA1 antibody (antiHu) that recognizes neuronal nuclei in the central and peripheral nervous
systems; ANNA1 is most commonly associated with small cell carcinoma of the lung

Correct Answer. b

(184). All are true about changes in pancreas in Diabetes mellitus except

a. Seen more with type I as compared to type II

b. Lymphocyte infiltrate is of B cell type

c. Amyloid deposition is more with type II

d. Reduction in number and size of islets

Solution. B
Lesions in the pancreas are inconstant and rarely of diagnostic value. Distinctive changes are more commonly associated with type 1 than
with type 2 diabetes. One or more of the following alterations may be present:
• Reduction in the number and size of islets. This is most often seen in type 1 diabetes, particularly with rapidly advancing disease. Most
of the islets are small and inconspicuous.
• Leukocytic infiltrates in the islets (insulitis) are principally composed of T lymphocytes, and are also seen in animal models of
autoimmune diabetes . Lymphocytic infiltrates may be present in type 1 diabetics at the time of clinical presentation. The distribution of
insulitis may be strikingly uneven in infants who fail to survive the immediate post natal period.
• In type 2 diabetes there may be a subtle reduction in islet cell mass, demonstrated only by special morphometric studies.
• Amyloid deposition within islets in type 2 diabetes begins in and around capillaries and between cells. At advanced stages, the islets
may be virtually obliterated; fibrosis may also be observed. Similar lesions may be found in older nondiabetics, apparently as part of
normal aging.

Correct Answer. b

Copyright © 2014 Delhi Academy of Medical Sciences, All Rights Reserved. 50/55
(185). Most common pituitary tumor in MEN-1 is

a. lactotroph adenoma

b. somatotroph adenoma

c. corticotroph adenoma

d. gonadotroph adenoma

Solution. A
The most frequent anterior pituitary tumor encountered in MEN-1 is a prolactinoma; some patients develop acromegaly from
somatotrophin-secreting tumors.

Correct Answer. a

(186). All are true about hairy leukoplakia except

a. EBV associated

b. Seen in immunodeficient states

c. Seen on the posterior part of the tongue

d. Balloon cells in the upper spinous layer

Solution. C
Hairy leukoplakia is a distinctive oral lesion on the lateral border of the tongue that is usually seen in immunocompromised patients and
is caused by Epstein-Barr virus (EBV). It can be observed in patients infected with the human immunodeficiency virus (HIV) and may
portend the development of AIDS. However, the lesions can also be found in patients who are immunocompromised for other reasons
including cancer therapy, transplant associated immunosuppression, and advancing age. Hairy leukoplakia takes the form of white,
confluent patches of fluffy (“hairy”), hyperkeratotic thickenings, almost always situated on the lateral border of the tongue. Unlike thrush,
the lesion cannot be scraped off. The distinctive microscopic appearance consists of hyperparakeratosis and acanthosis with “balloon
cells” in the upper spinous layer. Special stains can be used to demonstrate the presence of EBV RNA transcripts and proteins within the
lesional cells. Sometimes there is superimposed candidal infection on the surface of the lesions, adding to the “hairiness.”

Correct Answer. c

(187). Late genetic changes involved in transformation of Barret’s esophagus to adenocarcinoma are all except

a. CDKN2A

b. EGFR

c. MET

d. Cyclin D1

Solution. A
Molecular studies suggest that the progression of Barrett esophagus to adenocarcinoma occurs over an extended period through the
stepwise acquisition of genetic and epigenetic changes. This model is supported by the observation that epithelial clones identified in
nondysplastic Barrett metaplasia persist and accumulate mutations during progression to dysplasia and invasive carcinoma. Chromosomal
abnormalities, mutation of TP53, and downregulation of the cyclin-dependent kinase inhibitor CDKN2A, also known as p16/INK4a, are
detected at early stages. In the case of CDKN2A, both allelic loss and hypermethylation-induced epigenetic silencing have been described.
Later during progression there is amplification of EGFR, ERBB2, MET, cyclin D1, and cyclin E genes

Correct Answer. a

(188). All of the following are true about Menetrier disease except

a. Excessive secretion of transforming growth factor beta

b. Diffuse hyperplasia of foveolar epithelium

c. Risk of gastric adenocarcinoma is increased

d. Corkscrew like appearance of glands

Copyright © 2014 Delhi Academy of Medical Sciences, All Rights Reserved. 51/55
Solution. A
Ménétrier disease is a rare disorder associated with excessive secretion of transforming growth factor α (TGF-α). It is characterized by
diffuse hyperplasia of the foveolar epithelium of the body and fundus and hypoproteinemia due to protein-losing enteropathy. Secondary
symptoms, such as weight loss, diarrhea, and peripheral edema, are commonly present. Symptoms and pathologic features of Ménétrier
disease in children are similar to those in adults, but pediatric disease is usually self-limited and often follows a respiratory infection. Risk
of gastric adenocarcinoma is increased in adults with Ménétrier disease. Ménétrier disease is characterized by irregular enlargement of
the gastric rugae. Some areas may appear polypoid. Enlarged rugae are present in the body and fundus but the antrum is generally
spared. Histologically, the most characteristic feature is hyperplasia of foveolar mucous cells. The glands are elongated with a corkscrew-
like appearance and cystic dilation is common. Inflammation is usually modest, although some cases show marked intraepithelial
lymphocytosis. Diffuse or patchy glandular atrophy, evident as hypoplasia of parietal and chief cells, is typical.

Correct Answer. a

(189). Most specific marker of GIST is

a. CD 117

b. ckit

c. CD 34

d. DOG-1

Solution. D
A/B is the most sensitive marker for GIST

Correct Answer. d

(190). Terminal digestion in intestine is affected in all of the following malabsorptive syndromes except

a. Celiac disease

b. Whipple disease

c. Inflammatory bowel disease

d. Bacterial gastroenteritis

Solution. B

Correct Answer. b

(191). Heart failure cells can be stained specifically using

a. Reticulin

b. Oil red O

c. Perl’s

d. Perioidic acid Schiff

Solution. C
Heart failure cells are hemosiderin laden macrophages seen in lung as a response of congestive heart failure and hence stain positive for
Perl’s stain

Correct Answer. c

(192). Fibrinoid necrosis is classically associated with all of the following except

a. Malignant arteriosclerosis

b. Rheumatic heart disease

c. Takayasu arteritis

d. Polyarteritis nodosa

Solution. C
C is a granulomatous vasculitis.

Copyright © 2014 Delhi Academy of Medical Sciences, All Rights Reserved. 52/55
Correct Answer. c

(193). Alcohol induced cardiomyopathy most characteristically is

a. Hypertrophic

b. Restrictive

c. Dilated

d. Any of the above

Solution. C

Correct Answer. c

(194). Spider cells are seen in which cardiac tumor

a. Myxoma

b. Angiosarcoma

c. Rhabdomyosarcoma

d. Rhabdomyoma

Solution. D
Rhabdomyomas are gray-white myocardial masses that can be small or up to several centimeters in diameter. They are usually multiple
and involve the ventricles preferentially, protruding into the lumen. Microscopically, they are composed of bizarre, markedly enlarged
myocytes. Routine histologic processing often artifactually reduces the abundant cytoplasm to thin strands that stretch from the nucleus
to the surface membrane, an appearance referred to as “spider” cells.

Correct Answer. d

(195). Supravital staining is needed to visualise all of the following except

a. Heinz bodies

b. HbH inclusions

c. Howell Jolly bodies

d. Reticulocytes

Solution. C
C are DNA remnants easily visualised on routine Romanowsky stain

Correct Answer. c

(196). Follicular lymphoma involves which of the following genes

a. c-myc

b. Cyclin D1

c. SOX 11

d. BCL2

Solution. D
Follicular lymphoma involves t(14;18) leading to BCL2 overexpression.

Correct Answer. d

(197). Most common immunoglobulin secreted in plasma cell myeloma

a. IgA

b. IgM

Copyright © 2014 Delhi Academy of Medical Sciences, All Rights Reserved. 53/55
c. IgD

d. IgG

Solution. D
IgG is the most common Ig followed by IgA involved in plasma cell myeloma

Correct Answer. d

(198). Negative acute phase reactants are all except

a. Transferrin

b. Albumin

c. Transthyretin

d. Ceruloplasmin

Solution. D
C is also known as pre albumin.
D is a positive acute phase reactant in addition to fibrinogen, CRP, hepcidin etc

Correct Answer. d

(199). All of the following genetic changes in AML are favourable prognosis wise except

a. AML with t(8;21)

b. AML with RUNX1 mutations

c. AML with t(16;16)

d. AML with biallelic CEBPA mutations

Solution. B
As per WHO 2016, A, C and D show favourable prognosis.

Correct Answer. b

(200). Most important cytokine for acute phase response in acute inflammation is

a. IL-1

b. TNF alpha

c. IL-6

d. TGF beta

Solution. C
Acute-phase proteins are plasma proteins, mostly synthesized in the liver, whose plasma concentrations may increase several hundredfold
as part of the response to inflammatory stimuli. Three of the bestknown of these proteins are Creactive protein (CRP), fibrinogen, and
serum amyloid A (SAA) protein. Synthesis of these molecules in hepatocytes is stimulated by cytokines, especially IL6 (for CRP and
fibrinogen) and IL1 or TNF (for SAA).

Correct Answer. c
Test Answer
1.(d) 2.(b) 3.(c) 4.(c) 5.(d) 6.(b) 7.(d) 8.(c) 9.(a) 10.(b)

11.(c) 12.(a) 13.(a) 14.(d) 15.(a) 16.(b) 17.(a) 18.(b) 19.(d) 20.(c)

21.(b) 22.(c) 23.(b) 24.(d) 25.(d) 26.(a) 27.(c) 28.(a) 29.(d) 30.(d)

31.(a) 32.(c) 33.(d) 34.(b) 35.(d) 36.(d) 37.(c) 38.(b) 39.(a) 40.(d)

41.(c) 42.(a) 43.(c) 44.(d) 45.(b) 46.(c) 47.(a) 48.(d) 49.(d) 50.(c)

51.(b) 52.(a) 53.(a) 54.(c) 55.(a) 56.(a) 57.(d) 58.(c) 59.(d) 60.(c)

61.(b) 62.(d) 63.(b) 64.(d) 65.(a) 66.(c) 67.(a) 68.(b) 69.(a) 70.(c)

71.(a) 72.(d) 73.(b) 74.(b) 75.(d) 76.(c) 77.(a) 78.(c) 79.(a) 80.(b)

Copyright © 2014 Delhi Academy of Medical Sciences, All Rights Reserved. 54/55
81.(d) 82.(c) 83.(b) 84.(a) 85.(d) 86.(b) 87.(c) 88.(c) 89.(c) 90.(c)

91.(c) 92.(d) 93.(c) 94.(c) 95.(a) 96.(a) 97.(d) 98.(d) 99.(b) 100.(a)

101.(b) 102.(a) 103.(a) 104.(c) 105.(b) 106.(b) 107.(d) 108.(c) 109.(a) 110.(d)

111.(c) 112.(a) 113.(c) 114.(c) 115.(a) 116.(a) 117.(c) 118.(d) 119.(c) 120.(a)

121.(d) 122.(d) 123.(c) 124.(c) 125.(d) 126.(c) 127.(c) 128.(d) 129.(a) 130.(c)

131.(c) 132.(d) 133.(b) 134.(c) 135.(b) 136.(a) 137.(c) 138.(d) 139.(d) 140.(a)

141.(d) 142.(c) 143.(a) 144.(b) 145.(c) 146.(c) 147.(d) 148.(d) 149.(d) 150.(d)

151.(a) 152.(c) 153.(d) 154.(d) 155.(d) 156.(b) 157.(d) 158.(c) 159.(d) 160.(a)

161.(c) 162.(c) 163.(d) 164.(c) 165.(c) 166.(a) 167.(d) 168.(d) 169.(b) 170.(c)

171.(a) 172.(b) 173.(a) 174.(a) 175.(b) 176.(b) 177.(d) 178.(a) 179.(a) 180.(b)

181.(a) 182.(b) 183.(b) 184.(b) 185.(a) 186.(c) 187.(a) 188.(a) 189.(d) 190.(b)

191.(c) 192.(c) 193.(c) 194.(d) 195.(c) 196.(d) 197.(d) 198.(d) 199.(b) 200.(c)

Copyright © 2014 Delhi Academy of Medical Sciences, All Rights Reserved. 55/55

S-ar putea să vă placă și